Blackboard EMQs Mixed Flashcards
An unconscious 35-year-old man who has a capillary blood glucose of 1.5 mmol/L.
What would be the most appropriate next step? A. Gastric lavage B. Commence CPR C. Intravenous dextrose D. Inhaled anticholinergic E. CT scan brain F. IV antibiotics G. Endotracheal intubation H. Intravenous naloxone I. Precordial thump J. Intramuscular glucagon K. Lumbar puncture L. DC cardioversion
C. Intravenous dextrose
This patient is profoundly hypoglycaemic (symptoms of hypoglycaemia are present when glucose drops <3mmol/L). Patients with either long standing DM or on beta blockers may become unaware of hypoglycaemia and become profoundly hypoglycaemic before symptoms develop. In DM, hypoglycaemia is usually secondary to insulin or oral hypoglycaemics. Non DM causes of hypoglycaemia include insulinomas, alcohol, liver failure and Addison’s disease. Treatment is corrective and for someone this profoundly hypoglycaemic, IV dextrose is needed. Care should be taken when administering such high % dextrose IV due to the risk of skin necrosis if administered incorrectly (if the IV leaks). IM glucagon is only used if IV access cannot be established.
A 55-year-old man found collapsed at home who, on arrival at hospital, has no palpable pulse or recordable blood pressure.
What would be the most appropriate next step? A. Gastric lavage B. Commence CPR C. Intravenous dextrose D. Inhaled anticholinergic E. CT scan brain F. IV antibiotics G. Endotracheal intubation H. Intravenous naloxone I. Precordial thump J. Intramuscular glucagon K. Lumbar puncture L. DC cardioversion
B. Commence CPR
This patient has cardiac arrest and the first thing you need to do is commence CPR. It is thought to work by raising intrathoracic pressure and providing direct cardiac compression. 30 compressions (at 100/min) and 2 breaths for a total of 5 cycles (2 minutes), makes up one cycle of CPR. Compressions are the first priority, breaths may follow but compressions are paramount. Further management depends on whether the patient has a shockable rhythm (pulseless CT or VF) or non-shockable rhythm (PEA or asystole). Do you know when a precordial thump can be used?
An 18-year-old woman found unconscious at home. She has needle “track” marks in her arms, a respiratory rate of 10/min. and pinpoint pupils.
What would be the most appropriate next step? A. Gastric lavage B. Commence CPR C. Intravenous dextrose D. Inhaled anticholinergic E. CT scan brain F. IV antibiotics G. Endotracheal intubation H. Intravenous naloxone I. Precordial thump J. Intramuscular glucagon K. Lumbar puncture L. DC cardioversion
H. Intravenous naloxone
Opiate OD signs include CNS depression, miosis and apnoea. Finding small constricted pupils in someone who is unconscious is highly indicative. Naloxone is indicated both therapeutically and diagnostically. If there is a response, then it is diagnostic. Another diagnosis should be sought if the patient is unresponsive. IV is the preferred route of administration although naloxone can be given IM or SC if IV access cannot be established. Ventilatory support is key with 100% oxygen. You can check out Toxbase for a full database on poisons and treatments.
A 34-year-old woman who complained of a severe headache on waking and then collapsed.
What would be the most appropriate next step? A. Gastric lavage B. Commence CPR C. Intravenous dextrose D. Inhaled anticholinergic E. CT scan brain F. IV antibiotics G. Endotracheal intubation H. Intravenous naloxone I. Precordial thump J. Intramuscular glucagon K. Lumbar puncture L. DC cardioversion
E. CT scan brain
A CT head is indicated here in this possible SAH. This may show hyperdense areas in the basal cisterns, major fissures and sulci.
An 18-year-old known asthmatic with a respiratory rate of 50 and inaudible breath sounds on auscultation.
What would be the most appropriate next step? A. Gastric lavage B. Commence CPR C. Intravenous dextrose D. Inhaled anticholinergic E. CT scan brain F. IV antibiotics G. Endotracheal intubation H. Intravenous naloxone I. Precordial thump J. Intramuscular glucagon K. Lumbar puncture L. DC cardioversion
G. Endotracheal intubation
This is life threatening asthma that has not responded to therapy. There is cyanosis and respiratory acidosis despite the tachypnoea of 50/min. This patient is clearly too dyspnoeic to speak and oxygen saturation may be quite low. Pulsus paradoxus may also be observed. This patient should be intubated (mechanical ventilation is required) and transfered to ICU and given supplemental oxygen. IV corticosteroids should also be used and heliox therapy considered (helium-oxygen).
A 28-year-old female with recent onset of depression takes 50 capsules, 500 mg each of paracetamol tablets. In several days, the liver is most likely to show what?
A. Portal hypertension B. Hepatocellular carcinoma C. Cirrhosis D. Portal chronic inflammation E. Cholecystitis F. Hepatitis C virus infection G. Extensive necrosis H. Mallory weiss tear of oesophagus I. Hepatitis B virus infection
G. Extensive necrosis
Paracetamol OD can occur after a single large OD or repeated ODs. Often, the patient is asymptomatic at initial presentation but if untreated may cause liver injury over the 2-4 days after ingestion, including fulminant liver failure. Paracetamol is the most frequent intentional OD drug in this country. The risk of liver damage is increased after taking drugs which induce CYP 450. Inducers include St John’s wort, barbiturates, phenytoin, tetracycline, chronic alcohol use and carbamazepine. A serum paracetamol level is important to order as early as possible, but at the earliest 4 hours post-ingestion.Treatment if indicated is with N-acetylcysteine with the level based on a paracetamol treatment graph.
A 40-year-old male has a long history of chronic alcoholism. His liver is firm on palpation. An abdominal CT scan reveals that the liver has changes consistent with cirrhosis. He joins Alcoholics Anonymous and stops drinking. Despite his continued abstinence from alcohol, he remains at risk for development of which disease?
A. Portal hypertension C. Cirrhosis D. Portal chronic inflammation E. Cholecystitis F. Hepatitis C virus infection G. Extensive necrosis H. Mallory weiss tear of oesophagus I. Hepatitis B virus infection
B. Hepatocellular carcinoma
Patients with cirrhosis, especially those with alcoholic liver disease, are at a high risk of developing HCC. Cirrhosis is irreversible so despite stopping drinking, he is still at risk of HCC (hepatoma). Patients with cirrhosis should be screened for HCC with serum AFP and USS at 6 month intervals.
A 40-year-old female, rather overweight, has developed right upper quadrant pain and fever. What is the most likely diagnosis?
A. Portal hypertension B. Hepatocellular carcinoma C. Cirrhosis D. Portal chronic inflammation E. Cholecystitis F. Hepatitis C virus infection G. Extensive necrosis H. Mallory weiss tear of oesophagus I. Hepatitis B virus infection
E. Cholecystitis
Cholecystitis is acute GB inflammation caused by an obstruction at the cystic duct. It occurs as a major complication of gallstones and classically presents with RUQ pain and fever. Gallstones in EMQs classically involves the Fs (Fat, Forty, Female, Fertile, Fair). USS is the definitive initial investigation. HIDA scanning and MRI may help if the diagnosis remains unclear. Treatment is with cholecystectomy.
A 58-year-old man, who smoked 30 cigarettes a day, presents with a 6-week history of cough, malaise, anorexia and weight loss. Past medical history includes hypertension for which he has taken lisinopril and bendrofluazide for 4 years.
What is the most likely diagnosis? A. Carcinoma of bronchus B. Postnasal drip C. Foreign body D. Angiotensin converting enzyme inhibitor E. Oesophageal reflux F. Sarcoidosis G. COPD H. Tuberculosis I. Asthma J. Bronchiectasis
A. Carcinoma of bronchus
The history of smoking and weight loss point to a bronchial carcinoma. Initial investigation is with a CXR. Diagnosis relies on pathological confirmation from a tissue sample, often obtained from bronchoscopy. First line treatment aims at surgical resection if possible. Small cell lung cancer is treated with chemotherapy and is associated with SIADH and ectopic ACTH. Non-small cell lung cancer is more often associated with clubbing. Squamous cell carcinoma is associated with PTHrp release and is treated with radiotherapy. Adenocarcinomas are usually located peripherally in the lung and are more common in non-smokers although most cases are still associated with smoking. The paraneoplastic syndromes may include Lambert-Eaton myasthenic syndrome.
A 45-year-old woman who smokes 25 cigarettes a day is reviewed in the diabetic clinic. She has had a dry cough for 2 months. She is on numerous tablets as her diabetes is complicated by microalbuminuria and hypertension. Her GP had given her a course of antibiotics 2 weeks previously.
What is the most likely diagnosis? A. Carcinoma of bronchus B. Postnasal drip C. Foreign body D. Angiotensin converting enzyme inhibitor E. Oesophageal reflux F. Sarcoidosis G. COPD H. Tuberculosis I. Asthma J. Bronchiectasis
D. Angiotensin converting enzyme inhibitor
A dry cough is a side effect of ACE inhibitors due to the build up of bradykinin which is normally degraded by ACE. ARB such as losartan will be indicated in this case. ARBs are insurmountable antagonists of AT1 receptors for angiotensin II, preventing its renal and vascular effects.
A 40-year-old Afro-Caribbean woman presents with bilateral parotid swelling, and painful nodules on the front of the shins. She has a dry cough and slight shortness of breath on exertion.
What is the most likely diagnosis? A. Carcinoma of bronchus B. Postnasal drip C. Foreign body D. Angiotensin converting enzyme inhibitor E. Oesophageal reflux F. Sarcoidosis G. COPD H. Tuberculosis I. Asthma J. Bronchiectasis
F. Sarcoidosis
Sarcoidosis is a chronic multisystem disease with an unknown aetiology. The painful (mauve) nodules are erythema nodosum. Lupus pernio is another typical skin manifestation of sarcoidosis presenting with indurated plaques with discoloration on the face. Parotid enlargement is a classic feature (involvement of exocrine glands). The dry cough and SOB on exertion indicate pulmonary involvement.
CXR will typically show bilateral hilar lymphadenopathy and CXR findings are used in the staging of disease. Additionally, serum calcium and ACE levels may be raised. A transbronchial biopsy is essential for diagnosis in most cases and shows the presence of non-caseating granulomas. Black people have a higher lifetime risk of sarcoidosis, as do those of Scandinavian origin. The mainstay of treatment for severe disease involves systemic corticosteroids.
An 18-year-old man presents with a night-time cough and shortness of breath while playing football. This has got progressively worse over the previous 2 months.
What is the most likely diagnosis? A. Carcinoma of bronchus B. Postnasal drip C. Foreign body D. Angiotensin converting enzyme inhibitor E. Oesophageal reflux F. Sarcoidosis G. COPD H. Tuberculosis I. Asthma J. Bronchiectasis
I. Asthma
SOB and the cough, which may wake the patient from sleep combined with the patient’s age and progessive course suggest asthma. Examination can show an expiratory wheeze but may be normal and treatment is step-wise based on BTS guidelines. It is worth noting that in severe exacerbations, the chest may be silent. Night symptoms occur in more severe asthma and symptoms can be exacerbated by exercise. Diagnosis is supported by PEFR variation of at least 20% over 3 days in a week over several weeks or an increase of at least 20% to treatment. Look up the BTS guidelines for more information.
A 30-year-old man, a lifelong non-smoker, presents with a history of at least 6 months of purulent sputum. He has had regular chest infections since an attack of measles at the age of 14.
What is the most likely diagnosis? A. Carcinoma of bronchus B. Postnasal drip C. Foreign body D. Angiotensin converting enzyme inhibitor E. Oesophageal reflux F. Sarcoidosis G. COPD H. Tuberculosis I. Asthma J. Bronchiectasis
J. Bronchiectasis
Bronchiectasis is permanent bronchi dilatation due to bronchial wall damage and loss of elasticity. It is often as a consequence of recurrent/severe infections and most present with chronic productive mucopurulent cough. The most common identifiable cause is CF. Chest CT is the diagnostic test. Diagnosis is aided by sputum analysis. Have a think about what you would expect to hear on ascultation of the chest.
A 19-year-old student complaining of amenorrhoea for 9 months. Weight loss, generalised weakness and depression. She has started a vegan diet a year ago. Her BMI is 16.
What is the most likely diagnosis? A. Depression B. Hyperthyroidism C. Malignancy D. Cardiac failure E. Infestation with helminths F. Tuberculosis G. HIV H. Diabetes mellitus I. Liver failure J. Addison's disease K. Anorexia nervosa L. Renal failure M. Malabsorption
K. Anorexia nervosa
Anorexia nervosa often occurs in women which is a key risk factor. Puberty/adolescence and obsessive personality traits are additional risks. There is a higher incidence reported in western cultures (presumably skinny models on TV, adverts etc…) and studies on immigrants moving to a western culture exhibited a higher incidence. There is also postulated to be a genetic contribution from twin studies.Those who in higher socioeconomic classes are more affected. BMI is usually <17.5 and the patient often has a fear of weight gain and will refuse intervention to gain weight. The patient often fails to acknowledge how thin they are and can indicate how some parts of her body are ‘big’. Amenorrhoea is also a common complaint. Often anorexics eat vegetarian or vegan diets and you should inquire about the patient’s eating habits.There are two subtypes of AN. Restrictive where the patient will diet and exercise or the bingeing/purging types where there is also calorie restriction but ocassional binges and purging afterwards. There is also a tendency to laxative, diuretic and enema abuse.
A 17-year-old man returning from holiday in Africa. He presents with abdominal pain, weight loss, pruritis and a wheeze. Blood film showed eosinophilia.
What is the most likely diagnosis? A. Depression B. Hyperthyroidism C. Malignancy D. Cardiac failure E. Infestation with helminths F. Tuberculosis G. HIV H. Diabetes mellitus I. Liver failure J. Addison's disease K. Anorexia nervosa L. Renal failure M. Malabsorption
E. Infestation with helminths
The only option on the list that would give eosinophilia is infection with helminths. There is a risk factor here, having returned from Africa where the sanitation, hygiene and agricultural practices may leave much to be desired. Testing for stool ova and parasites will be needed to see exactly which helminth is causing this infection, although this is not very sensitive for strongyloides larvae. IgG serology can be used with >95% sensitivity if stool samples are negative in the case of strongyloides. This could be strongyloides or ascariasis or a rarer organism. Treatment will be with antihelminths depending on the organism. Ivermectin for strongyloides is the drug of choice. Other antihelmintic agents include albendazole, mebendazole and pyrantel pamoate.
A 70-year-old man with a history of 10kg weight loss over the previous 3 months. More recently, he developed acute lower back pain. He presents to A&E with coughing and sputum. Chest x-ray shows left lower lobe pneumonia.
What is the most likely diagnosis? A. Depression B. Hyperthyroidism C. Malignancy D. Cardiac failure E. Infestation with helminths F. Tuberculosis G. HIV H. Diabetes mellitus I. Liver failure J. Addison's disease K. Anorexia nervosa L. Renal failure M. Malabsorption
C. Malignancy
This person likely has lung cancer which has resulted in the significant weight loss of 10kg. This is post-obstructive pneymonia which is common in lung cancer patients and is caused, most of the time, by a large and centrally obstructing tumour. It is essential to relieve this obstruction in this case and many techniques can be tried. There is also bone pain here in the lower spinal column which is due to metastases. The prognosis here is not good.
A 25-year-old woman with fatigue and weight loss. She gives a history of frequent loose stools with abdominal pain. Full blood count revealed iron deficiency anaemia.
What is the most likely diagnosis? A. Depression B. Hyperthyroidism C. Malignancy D. Cardiac failure E. Infestation with helminths F. Tuberculosis G. HIV H. Diabetes mellitus I. Liver failure J. Addison's disease K. Anorexia nervosa L. Renal failure M. Malabsorption
M. Malabsorption
There is frequent loose stools here and abdominal pain. Combined with the IDA, this points to malabsorption. This could well be a presentation of coeliac disease – IDA is one of the most common clinical presentations and abdominal pain and diarrhoea are common. Coeliac disease is a systemic autoimmune condition triggered by dietary gluten peptides found in grains. It is a relatively common condition. The only treatment is a strict gluten-free diet for life.
A 40-year-old African refugee has noticed recent weight loss. Although he attributed this to stress you are concerned when you detect generalised lymphadenopathy. Blood count shows neutropenia and thrombocytopenia.
What is the most likely diagnosis? A. Depression B. Hyperthyroidism C. Malignancy D. Cardiac failure E. Infestation with helminths F. Tuberculosis G. HIV H. Diabetes mellitus I. Liver failure J. Addison's disease K. Anorexia nervosa L. Renal failure M. Malabsorption
G. HIV
HIV is a retrovirus and there are two types, HIV 1 which is the main virus responsible and HIV 2 which is restricted to parts of West Africa. Weight loss is common in HIV and if more than 10% body weight is lost of BMI reduces to 18.5, this is an indication of more severe immunocompromise. Weight loss in HIV may result from malnutrition, co-existent TB infection or HIV wasting syndrome, the latter being an AIDS defining illness. Generalised lymphadenopathy is also common and is characterised by the painless enlargement of 2 more more non-contiguous sites of >1cm for >3 months. Neutropenia is also seen due to CD4 deficiency and thrombocytopenia may also be seen along with an anaemic picture.
There are WHO (stage 1-4) and CDC criteria used in clinical staging. This patient needs to have a CD4 count, HBV and HCV screen, VDRL (syphilis), tuberculin skin test (TB) and CXR. HIV viral load will also be assessed. Prophylaxis and immunisations should be considered against infections such as hepatitis, influenza, PCP and TB. When to initiate HAART depends on the clinical stage, CD4 and co-morbidities. This patient will need to be started on HAART. Classes of antiretrovirals include NRTIs, NNRTIs, protease inhibitors, fusion inhibitors and integrase inhibitors.
A 25-year-old woman returning from Australia presents with acutely painful left calf. Ultrasound confirms deep vein thrombosis extending above the popliteal veins. She has recently missed a period.
What is the most appropriate management? A. Embolectomy B. Subcutaneous low molecular weight heparin C. Reassure and discharge D. Anti-embolism stocking E. Fondaparinux (FXa inhibitor) F. Intravenous heparin G. Start warfarin therapy H. Subcutaneous low molecular weight heparin I. Check INR and continue warfarin J. Vena cava filter K. Observation in hospital
B. Subcutaneous low molecular weight heparin
Women developing a DVT during pregnancy can be treated with heparin or LMWH. However, LMWH is preferred due to more dependable pharmacokinetics. The agents you will commonly hear include enoxaparin, dalteparin and tinzaparin.
You may see patients treated with unfractionated heparin instead of LMWH - this is done for patients at a higher bleeding risk because unfractionated heparin is more easily reversible with protamine.
A 30-year-old man developed acute pain in the right calf after a game of squash. He has marked calf tenderness but no swelling. Doppler ultrasound is negative.
What is the most appropriate management? A. Embolectomy B. Subcutaneous low molecular weight heparin C. Reassure and discharge D. Anti-embolism stocking E. Fondaparinux (FXa inhibitor) F. Intravenous heparin G. Start warfarin therapy H. Subcutaneous low molecular weight heparin I. Check INR and continue warfarin J. Vena cava filter K. Observation in hospital
C. Reassure and discharge
This is likely to be a musculoskeletal injury in an otherwise fit, young man. Eccentric contraction causes more frequent injury. Inadequate warm-up before exercise can predispose to muscle injuries.
A 50-year-old woman taking non-steroidal anti-inflammatory drugs for arthritis presented with a history of sudden onset pain behind her right knee leading to pain down the calf. Ultrasound confirms a Baker’s cyst.
What is the most appropriate management? A. Embolectomy B. Subcutaneous low molecular weight heparin C. Reassure and discharge D. Anti-embolism stocking E. Fondaparinux (FXa inhibitor) F. Intravenous heparin G. Start warfarin therapy H. Subcutaneous low molecular weight heparin I. Check INR and continue warfarin J. Vena cava filter K. Observation in hospital
C. Reassure and discharge
A popliteal cyst is an accumulation of synovial fluid which in this case has resulted from this woman’s arthritis. This is an accumulation of synovial fluid behind the knee, usually in response to injury or inflammation. It will self-resolve but the underlying cause should be addressed i.e. arthritis. First line treatment for grade 1 or 2 injuries is with RICE: rest, ice, compression and elevation followed by gentle mobilisation. Adjunctive analgesia can be offered with paracetamol. Treatment is conservative, particularly if asymptomatic. Surgery is only indicated in those with extensive symptoms where conservative and percutaneous treatments have failed. Corticosteroid injections (intra-articular) can also be considered.
You are asked to see a patient with acute chest pain 5 days after total hip replacement. BP 120/80, HR 93. A PE is confirmed. The patient has a previous history of heparin-induced thrombocytopenia.
What is the most appropriate management? A. Embolectomy B. Subcutaneous low molecular weight heparin C. Reassure and discharge D. Anti-embolism stocking E. Fondaparinux (FXa inhibitor) F. Intravenous heparin G. Start warfarin therapy H. Subcutaneous low molecular weight heparin I. Check INR and continue warfarin J. Vena cava filter K. Observation in hospital
E. Fondaparinux (FXa inhibitor)
A factor Xa antagonist is preferred if the patient has or has had heparin-induced thrombocytopenia. If the patient has a low BP then systemic thrombolysis would be indicated to prevent possible cardiac arrest.
A woman who is on warfarin for a confirmed right calf DVT develops increasing pain and swelling of that leg. This is the third time this has happened. Repeat imaging shows thrombus limited to the calf.
What is the most appropriate management? A. Embolectomy B. Subcutaneous low molecular weight heparin C. Reassure and discharge D. Anti-embolism stocking E. Fondaparinux (FXa inhibitor) F. Intravenous heparin G. Start warfarin therapy H. Subcutaneous low molecular weight heparin I. Check INR and continue warfarin J. Vena cava filter K. Observation in hospital
H. Subcutaneous low molecular weight heparin
Patients with recurrent thromboembolism despite on warfarin should be given heparin or LMWH. LMWH is again the primary option for reasons discussed. They should be given for at least 5 days until INR is between 2 and 3 (the target INR). Fondaparinux would be used instead if there was a high chance of delayed HIT. Warfarin is continued and efforts must be made to work out how this patient has developed a DVT despite on warfarin therapy. There may be subtherapeutic dosing, the presence of a malignancy or antiphospholipid syndrome. If there is documented thrombosis progression despite adequate anticoagulation, without HIT and other causes are excluded, an IVC filtre may be indicated but evidence of its efficacy have been debated by recent studies.
A 40 year old lady with multiple painful lumps in her breast, which are painful & tender pre-menstrually.
What is the most likely diagnosis? A. Duct Ectasia B. Lipoma C. Sebaceous cyst D. Carcinoma of the breast E. Gynaecomastia F. Fibroadenoma G. Breast Abscess I. Breast Cyst
H. Fibroadenosis
Fibrocystic breasts are characterised by ‘lumpy’ breasts associated with pain which fluctuates with the menstrual cycle (it is worse during the luteal phase of menses). Risk factors include obesity, nulliparity, HRT and late onset menopause and first childbirth. It is a diagnosis of exclusion, and is considered to be an exaggerated physiological phenomenon rather than a disease (54% of clinically normal breasts are found on autopsy to have fibrocystic changes). Symptoms typically arise between the 3rd and 4th decases of life. There may also be a nipple discharge, which can be suspicious if bloody or profuse etc and may indicate the presence of an intraductal papilloma, cancer, or duct ectasia. Cysts can be aspirated if symptomatic (asymptomatic or small ones do not require intervention). If the aspirate is straw coloured and completely aspirated, there is no need for cytology, but if the aspirate is bloody, cytology or biopsy is needed to exclude cancer. There is improvement of mastalgia and cysts at menopause and until then it runs a chronic relapsing course.
A 25 year old lady with a discrete, non-tender, mobile lump in one breast.
What is the most likely diagnosis? A. Duct Ectasia B. Lipoma C. Sebaceous cyst D. Carcinoma of the breast E. Gynaecomastia F. Fibroadenoma G. Breast Abscess H. Fibroadenosis I. Breast Cyst
F. Fibroadenoma
This is a fibroadenoma which tends to be asymptomatic and found incidentally, typically in a patient <40 years old. It is a mobile mass (sometimes called breast mice), firm, painless and can also be described as smooth and rubbery. It is benign with epithelial and stromal elements.
A 35 year old lady is generally unwell with a tachycardia & a fever. A segment of the right breast is painful, tender, red & warm
What is the most likely diagnosis? A. Duct Ectasia B. Lipoma C. Sebaceous cyst D. Carcinoma of the breast E. Gynaecomastia F. Fibroadenoma G. Breast Abscess H. Fibroadenosis I. Breast Cyst
G. Breast Abscess
Breast abscess presents with mastalgia and fever. Breast infection typically affects women who are lactating and the most commonly implicated pathogen is staphylococcus aureus. The painful, hard and red lump indicates the development of an abscess. Antibiotic therapy is indicated with surgical intervention such as aspiration and drainage with possible duct excision. Prompt management of mastitis when it presents will usually lead to a good timely resolution and prevent the development of complications such as an abscess. An USS can help to identify the underlying abscess which usually forms a hypoechoic lesion. Needle aspiration can be used both therapeutically and diagnostically and can be guided by ultrasound.
A 14 year old boy with bilateral breast enlargement.
What is the most likely diagnosis? A. Duct Ectasia B. Lipoma C. Sebaceous cyst D. Carcinoma of the breast E. Gynaecomastia F. Fibroadenoma G. Breast Abscess H. Fibroadenosis I. Breast Cyst
E. Gynaecomastia
This is a boy who has enlarged breasts. Normal to see gynaecomastia in puberty. Other causes include liver disease and as a side effect of drugs such as digoxin, spironolactone and cimetidine.
A 40 year old lady with a green nipple discharge & tender lumpiness beneath the areola.
What is the most likely diagnosis? A. Duct Ectasia B. Lipoma C. Sebaceous cyst D. Carcinoma of the breast E. Gynaecomastia F. Fibroadenoma G. Breast Abscess H. Fibroadenosis I. Breast Cyst
A. Duct Ectasia
Duct ectasia happens because the lactiferous duct gets blocked. Ectasia means widening. It can mimic breast cancer as the discharge can be bloody sometimes and signs can include nipple inversion. The green nipple discharge is typical in EMQs. It is a self limiting condition.
A 60 year old man with a history of occupational exposure in building & demolition industry presents with shortness of breath. On examination there are signs of a pleural effusion and the patient is clubbed.
What would be the most appropriate investigation? A. Colonoscopy B. Bronchoscopy C. Sputum culture D. Abdominal ultrasound E. Echocardiogram F. Stool culture G. Lung funtion tests H. Chest x-ray
H. Chest x-ray
Idiopathic pulmonary fibrosis (previously known as Cryptogenic fibrosing alveolitis) progresses over several years and is characterised by pulmonary scar tissue formation and dyspnoea. Patients complain of a non-productive cough and typically reproducible and predictable SOB on exertion. This man’s work means he comes into contact with small organic or inorganic dust particles which is thought to be implicated in the cascade of events leading to IPF. Another risk factor is cigarette smoking which significantly increases the risk of IPF. The mean age of diagnosis is 60-70. CXR here will show reticular opacities. A high resolution CT scan can also be done if it was an option on this list.
A 19 year old woman with a past history of cardiac surgery in infancy presents with symptoms of decreasing exercise tolerance. On examination there is cyanosis & clubbing.
What would be the most appropriate investigation? A. Colonoscopy B. Bronchoscopy C. Sputum culture D. Abdominal ultrasound E. Echocardiogram F. Stool culture G. Lung funtion tests H. Chest x-ray
E. Echocardiogram
The only cardiac investigation here is an echocardiogram. Cardiovascular causes of clubbing include cyanotic congenital heart disease, infective endocarditis and atrial myxoma. This could well be Eisenmenger’s syndrome caused by shunt reversal.
A 35 year old woman with history of recurrent lower abdominal pain, bloody diarrhoea & passing mucus PR. On examination there is lower abdominal tenderness & clubbing.
What would be the most appropriate investigation? A. Colonoscopy B. Bronchoscopy C. Sputum culture D. Abdominal ultrasound E. Echocardiogram F. Stool culture G. Lung funtion tests H. Chest x-ray
A. Colonoscopy
This woman has inflammatory bowel disease (which is a cause of clubbing), which by the history is more likely to be UC than CD, whereby the mainstay of treatment is with 5-ASA. A colonoscopy is required to assess the extent of disease and for a definitive diagnosis. Biopsy in CD will show transmural granulomatous inflammation. CD can affect the whole GIT but favours the TI and proximal colon and is macroscopically characterised by skip lesions. UC on the other hand is characterised by the presence of crypt abscesses, which is pathognomic. CD risk is increased 3-4 fold by smoking whereas smoking seems protective in UC. The mainstay of treatment in CD is with steroids and azathioprine to revent relapses and for those suffering side effects of steroid treatment. TNF-alpha inhibitors also have a role. Surgery in CD is only indicated in a small number of patients who bleed, for bowel perforation and cases of complete obstruction. The aim is to rest distal disease by temporarily diverting faecal flow.While a stool culture may be useful in ruling out infectious gastroenteritis it will not provide the diagnosis here.
A 50 year old woman who is a heavy smoker presents with shortness of breath & weight loss. On examination she is clubbed. The chest x-ray shows a perihilar shadow.
What would be the most appropriate investigation? A. Colonoscopy B. Bronchoscopy C. Sputum culture D. Abdominal ultrasound E. Echocardiogram F. Stool culture G. Lung funtion tests H. Chest x-ray
B. Bronchoscopy
The history of smoking and weight loss point to a bronchial carcinoma. Whilst the initial investigation is with a CXR, diagnosis relies on pathological confirmation from a tissue sample, often obtained from bronchoscopy. Non-small cell lung cancer is more often associated with clubbing.
A 37 year old Somali doctor with a history of coughing up about half a cup of fresh blood on 4-5 occasions over the period of 2 months. He had lost 5kg in weight and had drenching night sweats almost daily. Chest x-ray shows a large cavitating nodule in the left upper zone.
What is the most likely diagnosis? A. Pulmonary aspergillosis B. Thrombocytopenia C. Pulmonary embolus D. Arterio-venous malformation E. Asthma F. Bronchiectasis G. Tuberculosis H. Sickle cell crisis I. Acute left ventricular failure J. Carcinoma of bronchus K. Inhaled foreign body
G. Tuberculosis
This patient has pulmonary TB. Risk factors include HIV infection, exposure to infection and returning from or being born in a high risk region such as Somalia, or other areas of Africa, Asia and Latin America. The night sweats here, weight loss and haemoptysis are all suggestive. The CXR finding of a cavitating lesion and upper zone changes are also consistent with TB. If TB is suspected, the patient should be placed in isolation and 3 sputum samples cultured for AFB being the gold standard of diagnosis. Culture takes several weeks so sputum smears will be done before culture results are known. Interferon-gamma release assays (IGRAs) are now used by some hospitals to rapidly determine a patient’s TB status. All patients who have TB should be tested for HIV within 2 months of diagnosis.
A 55 year old lorry driver, a ‘fit’ smoker of 20-30 cigarettes a day, presents to his GP with a history of coughing up 2 streaks of blood on separate mornings in the previous week. He has had a morning cough with small amounts of sputum over the previous 10 years and admits on questioning that he may have lost weight and has had to tighten his trouser belt. The ESR was 75 and chest x-ray showed collapse of the right middle lobe.
What is the most likely diagnosis? A. Pulmonary aspergillosis B. Thrombocytopenia C. Pulmonary embolus D. Arterio-venous malformation E. Asthma F. Bronchiectasis G. Tuberculosis H. Sickle cell crisis I. Acute left ventricular failure J. Carcinoma of bronchus K. Inhaled foreign body
J. Carcinoma of bronchus
The weight loss, respiratory symptoms of haemoptysis and the history of smoking makes this likely to be bronchial carcinoma. Lobar collapse here may be a consequence of obstruction. First line treatment aims at surgical resection if possible. Small cell lung cancer is treated with chemotherapy and is associated with SIADH and ectopic ACTH. Non-small cell lung cancer is more often associated with clubbing. Squamous cell carcinoma is associated with PTHrp release and is treated with radiotherapy. Adenocarcinomas are usually located peripherally in the lung and are more common in non-smokers although most cases are still associated with smoking. The paraneoplastic syndromes may include Lambert-Eaton myasthenic syndrome.
A 50 year old cleaner, with a history of pertussis as a child, had a fever and increasing shortness of breath with sweating on exertion. She has had a cough productive of yellow sputum for 25 years but this is recently blood stained. She is a non-smoker. She has come to her GP for a further course of antibiotics and he refers her for a chest X-ray because he thinks she is clubbed.
What is the most likely diagnosis? A. Pulmonary aspergillosis B. Thrombocytopenia C. Pulmonary embolus D. Arterio-venous malformation E. Asthma F. Bronchiectasis G. Tuberculosis H. Sickle cell crisis I. Acute left ventricular failure J. Carcinoma of bronchus K. Inhaled foreign body
F. Bronchiectasis
Bronchiectasis is permanent bronchi dilatation due to bronchial wall damage and loss of elasticity. It is often as a consequence of recurrent/severe infections and most present with chronic productive mucopurulent cough. The most common identifiable cause is CF but this history here of pertussis is responsible in this instance. Chest CT is the diagnostic test. Diagnosis is aided by sputum analysis. Not only should the GP have this diagnosis in mind but should know whether the patient is actually clubbed, instead of ‘thinking’ she is clubbed. Bronchiectasis is a cause of clubbing.
A 25 year old public relations executive has gained weight on the combined oral contraceptive and is reluctant to give up smoking 10-15 cigarettes a day. She presents at an A&E department with acute left sided pleuritic chest pain, slight shortness of breath and a cough with a little sputum containing smears of blood. Chest X-ray is unremarkable.
What is the most likely diagnosis? A. Pulmonary aspergillosis B. Thrombocytopenia C. Pulmonary embolus D. Arterio-venous malformation E. Asthma F. Bronchiectasis G. Tuberculosis H. Sickle cell crisis I. Acute left ventricular failure J. Carcinoma of bronchus K. Inhaled foreign body
C. Pulmonary embolus
The underlying pathophysiology of PE is based on Virchow’s triad. SOB is a common symptom and there may also be pleuritic chest pain and haemoptysis. Strong risk factors include recent surgery, DVT, obesity, prolonged bed rest, malignancy, previous VTE, pregnancy and the thrombophilias such as factor V Leiden. The oral contraceptive pill is associated with an increased risk of VTE but is a weak risk factor. However, the rest of this history still makes the diagnosis in this case. CXR may be normal like in this case or may have findings suggestive of PE such as band atelectasis, hemidiaphragm elevation, Fleischner’s sign, Westermark’s sign and Hampton hump. ECG may be normal, or may show tachycardia, new RAD, new RBBB or the classical S wave in I, Q wave with T inversion in III. Various clinical probability scores exist for PE and D-dimer can be used to exclude PE as a diagnosis.
A 45 year old woman who is HIV positive reports easy bruising, frequent nose bleeds and coughing up small streaks of blood. She is otherwise well at present. Chest X-ray shows no abnormality. A full blood count shows Hb of 10.5g/dl, WBC 5.0 x 109/l and platelet count 28 x 109/l.
What is the most likely diagnosis? A. Pulmonary aspergillosis B. Thrombocytopenia C. Pulmonary embolus D. Arterio-venous malformation E. Asthma F. Bronchiectasis G. Tuberculosis H. Sickle cell crisis I. Acute left ventricular failure J. Carcinoma of bronchus K. Inhaled foreign body
B. Thrombocytopenia
HIV is implicated in idiopathic thrombocytopenic purpura, which is a condition of abnormally low platelet count of unknown cause. This question makes this dead easy by giving you the platelet count as 28 x109/L and all you need to do is to appreciate that a count under 150 x109/L is defined as thrombocytopenia. As well as the count, the patient has obvious symptoms of thrombocytopenia here with bruising, haemoptysis and epistaxis.
A 30 year old man appears jaundiced. FBC and peripheral blood smear is reported as normal. INR is normal. LFTs show elevated unconjugated bilirubin. Liver enzymes are normal.
What is the most likely diagnosis? A. Liver secondaries B. Primary biliary cirrhosis C. Alcoholic cirrhosis D. Gilbert’s syndrome E. Paracetamol poisoning F. Carcinoma of head of pancreas G. Autoimmune haemolytic anaemia H. Cholelithiasis I. Viral hepatitis A
D. Gilbert’s syndrome
Gilbert’s occurs in an asymptomatic patient, often as an incidental finding or mild jaundice occuring in adolescence/young adult age. There is elevated unconjugated BR with other liver tests being normal. The blood smear is also normal with normal reticulocyte count, and normal Hb indicating that this is not due to haemolysis. It is a common syndrome and is not really a disease, more a physiological variant. No treatment is needed and this condition is due to decreased UDPGT activity leading to decreased conjugation of unconjugated bilirubin, leading to elevated levels.
A 20 year old woman complains of breathlessness and appears pale. Hb 7.3 g/dl, WBC 7.0 x 109/L, Platelets 100 x 109/l. Unconjugated bilirubin is elevated. Direct antiglobulin test is positive.
What is the most likely diagnosis? A. Liver secondaries B. Primary biliary cirrhosis C. Alcoholic cirrhosis D. Gilbert’s syndrome E. Paracetamol poisoning F. Carcinoma of head of pancreas G. Autoimmune haemolytic anaemia H. Cholelithiasis I. Viral hepatitis A
G. Autoimmune haemolytic anaemia
This woman is complaining of symptoms of anaemia with SOB and pallor. Haemolytic anaemia can be either hereditary or acquired. Hereditary can be divided into 3 broad groups – either inherited defects in the membrane such as hereditary spherocytosis, enzyme deficiencies such as G6PDH deficiency or abnormal Hb production such as that seen in sickle cell anaemia and thalassaemia. Acquired can be either immune or non-immune in cause. Autoimmune HA here is due to autoantibodies, which occurs most often as part of another autoimmune process like SLE or RA or related to a lymphoproliferative disorder such as CLL. FBC here shows a low Hb consistent with the diagnosis, though it is important here to also request MCHC and reticulocyte count. Both of these would be increased in haemolytic anaemia. Increased breakdown of haem leads to an unconjugated hyperbilirubinaemia, though not >70-85, which would indicate some degree of liver impairment as well. LDH would also be raised, which can be helpful if there is no concurrent tissue damage. Haptoglobin (which binds free Hb) will also be low, and these two markers combined is 90% specific for HA. The giveaway for AIHA here is the positive DAT or Coombs’ test. This test detects IgG or complement bound to RBCs (hence a positive result suggests an immune cause of HA). The patient’s RBCs are washed and mixed with antiserum for IgG and C3d. The presence of IgG often indicates the presence of a warm antibody, whereas C3d suggests a cold antibody.
A 55 year old man complains of abdominal distension. Hb 10.0 g/dl, WBC 7.0 x 109/l, Platelets 100 x 109/l. The patient is hyponatraemic and hypokalaemic. Urea and creatinine are normal. Serum albumin is low. AST is elevated at 250 and ALT is elevated to 125. ALP and GGT are also elevated.
What is the most likely diagnosis? A. Liver secondaries B. Primary biliary cirrhosis C. Alcoholic cirrhosis D. Gilbert’s syndrome E. Paracetamol poisoning F. Carcinoma of head of pancreas G. Autoimmune haemolytic anaemia H. Cholelithiasis I. Viral hepatitis A
C. Alcoholic cirrhosis
This is a case of alcoholic cirrhosis. AST and ALT are elevated in all forms of alcoholic liver disease, both to <300. The true upper limit of normal AST and ALT can be considered to be 30 units/L for men and 19 units/L for women. The AST/ALT ratio is classically >2, which is seen in about 70% of cases, and AST is usually above ALT level. The elevated ALP here may represent cholestasis. GGT is a more sensitive marker than AST or ALT for heavy alcohol use and elevation here suggests the cause is alcohol. The anaemia here in alcoholic liver disease can be due to many causes like iron deficiency, folate deficiency and hypersplenism. Thrombocytopenia may be secondary to alcohol induced BM suppression, folate deficiency or hypersplenism. Those with advanced cirrhosis frequently are also hyponatraemic and hypokalaemia is also seen. Urea and creatinine is normal in this patient but elevation may suggest the presence of hepatorenal syndrome. Cirrhosis is the end stage of chronic liver disease and the abdominal distension may be a sign of ascites.
A 30 year old man who has returned from a month-long business trip to India has been complaining of abdominal pain and nausea for a week. He has also vomitied and remembers having a meal of shellfish from a street vendor. He woke up this morning and noticed he was going yellow. His ALT is 5000 and total bilirubin 139.
What is the most likely diagnosis? A. Liver secondaries B. Primary biliary cirrhosis C. Alcoholic cirrhosis D. Gilbert’s syndrome E. Paracetamol poisoning F. Carcinoma of head of pancreas G. Autoimmune haemolytic anaemia H. Cholelithiasis I. Viral hepatitis A
I. Viral hepatitis A
Hepatitis A is primarily transmitted via the faecal-oral route. After the virus is consumed and absorbed, it replicates in the liver and is excreted in the bile (to be re-transmitted). Transmission usually precedes symptoms by about 2 weeks and patients are non-infectious 1 week after onset of jaundice. The history can reveal risk factors such as living in an endemic area, contact with an infected person, homosexual sex or a known food-borne outbreak. This is classically, in EMQs, associated with shellfish which is harvested from sewage contaminated water. If the patient has other liver diseases such as HBV or HCV or cirrhosis then there is a higher risk of fulminant HAV infection. The clinical course of HAV consists of a pre-icteric phase, lasting 5-7 days, consisting characteristically of N&V, abdominal pain, fever, malaise and headache. Rarer symptoms may be present such as arthralgias and even severe thrombocytopenia and signs that may be found include splenomegaly, RUQ tenderness and tender hepatomegaly as well as bradycardia. The icteric phase is characterised by dark urine, pale stools, jaundice and pruritis. When jaundice comes on, the pre-icteric phase symptoms usually diminish, and jaundice typically peaks at 2 weeks. However, a fulminant course runs in <1% of patients with worsenining jaundice and encephalopathy. Serum transaminases may reach in excess of 10,000 units, although there is little correlation between the level and disease severity. ALT is typically higher than AST.
A 45 year old obese woman complains of intermittent abdominal pain with jaundice. Hb 13.1 g/dl, WBC 8.3 x 109/l, Platelets 200 x 109/l, Na 140, K 4.1, Urea 5.2, Creatinine 88, ALP 580, Bilirubin 264, ALT 70
What is the most likely diagnosis? A. Liver secondaries B. Primary biliary cirrhosis C. Alcoholic cirrhosis D. Gilbert’s syndrome E. Paracetamol poisoning F. Carcinoma of head of pancreas G. Autoimmune haemolytic anaemia H. Cholelithiasis I. Viral hepatitis A
H. Cholelithiasis
Gallstones, or cholelithiasis, is highly prevalent but most are asymptomatic. Symptoms occur when the cystic or bile duct get obstructed or when the gallstones erode through the gallbladder. The biliary pain here is caused by transient cystic duct obstruction or passage/obstruction through the bile duct. The typical presentation is of RUQ or epigastric pain often after food which increases in intensity and lasts for several hours. The jaundice here suggests choledocholithiasis (the presence of at least one gallstone in the CBD). Gallstones in EMQs classically involves the Fs (Fat, Forty, Female, Fertile, Fair) which reflects the risk factors such as older age, female sex, obesity, rapid weight loss, pregnancy and drugs. USS is the definitive initial investigation. HIDA scanning and MRI may help if the diagnosis remains unclear. Definitive treatment is with laparoscopic cholecystectomy for those with symptoms. Complications of gallstones include cholecystitis, cholangitis and pancreatitis and symptoms may indeed overlap between these conditions.
A 45 year old woman from Jamaica presents with a 6 month history of weight loss and a two week history of fever. On examination she had cervical lymphadenopathy. Her calcium was raised at 3.0 and CXR showed bilateral hilar lymphadenopathy.
What is the most likely diagnosis? A. Post immunisation B. Malaria C. HIV infection D. Hodgkin’s lymphoma E. Appendicitis F. Systemic lupus erythematosis G. Pyelonephritis H. Sarcoidosis I. Drug reaction J. Influenza K. Glandular fever L. Gastric carcinoma M. Pneumonia N. Tuberculosis
H. Sarcoidosis
Sarcoidosis is a chronic multisystem disease with an unknown aetiology. Lymphadenopathy is a common presentation and nodes are enlarged but non-tender, typically involving the cervical and submandibular nodes. Although uncommon, the patient may present with unexplained modest weight loss (which is often mistaken for TB or lymphoma along with the abnormal CXR) and a low-grade fever. CXR will typically show bilateral hilar lymphadenopathy and CXR findings are used in the staging of disease. Additionally, serum calcium and ACE levels may be raised. A transbronchial biopsy is essential for diagnosis in most cases and shows the presence of non-caseating granulomas. Black people have a higher lifetime risk of sarcoidosis, as do those of Scandinavian origin. The mainstay of treatment for severe disease involves systemic corticosteroids. Skin manifestations include erythema nodosum which are tender erythematous nodules and lupus pernio presenting with indurated plaques with discoloration on the face.
A 25 year old man with a 3 day history of high temperatures, aching limbs and neck discomfort. Apart from temperature of 39℃and some mild conjunctivitis, examination was normal. Antibiotics were prescribed but had had no effect.
What is the most likely diagnosis? A. Post immunisation B. Malaria C. HIV infection D. Hodgkin’s lymphoma E. Appendicitis F. Systemic lupus erythematosis G. Pyelonephritis H. Sarcoidosis I. Drug reaction J. Influenza K. Glandular fever L. Gastric carcinoma M. Pneumonia N. Tuberculosis
J. Influenza
This is an acute respiratory tract infection caused by seasonal viral influenza A or B, hence antibiotics would have no effect. It is characterised by respiratory symptoms including rhinorrhoea, cough, fever, chills, headache and myalgia. Recent upper respiratory tract infection has led to associated viral conjuncitivitis. The examination in this case is of course otherwise unremarkable. Antigenic change presents a challenge in creating new vaccines as there is no incremental protection from previous vaccinations. There have been 4 pandemics since 1918, the recent one being ‘swine flu’ in April 2009, caused by new gene rearrangement of human, avian and swine influenza.
A 50 year woman normally resident in the UK returned from visiting relatives in Pakistan. She described intermittent fevers with rigors, diarrhoea and severe headaches. She is mildly jaundiced.
What is the most likely diagnosis? A. Post immunisation B. Malaria C. HIV infection D. Hodgkin’s lymphoma E. Appendicitis F. Systemic lupus erythematosis G. Pyelonephritis H. Sarcoidosis I. Drug reaction J. Influenza K. Glandular fever L. Gastric carcinoma M. Pneumonia N. Tuberculosis
B. Malaria
In the Western world, almost all cases of malaria occurs in travellers so an adequate travel history is crucial or the diagnosis may be missed. Patients typically present with non-specific symptoms such as a fever, sweats, chills and myalgia. This can also include the diarrhoea which this patient is experiencing and headaches. The jaundice here suggests falciparum infection, which is always the cause in severe disease. This woman has just returned from an endemic area. Sometimes EMQs will describe patterns of fevers occuring at regular intervals of 48-72 hours associated with P. vivax, P. ovale and P. malariae infections but in most patients there is no specific pattern. Hepatosplenomegaly is a common presenting sign although not common at presentation in a first world setting. Thrombocytopenia is common with falciparum infection and a mild degree of anaemia ais commonly seen. WCC can be high, low or normal. The severity of malaria depends partly on the species and also on host immunity. Therefore those who live in endemic areas may develop minimal symptoms due to IgG antibody and cell-mediated immunity and physiological tolerance of parasitaemia. Pregnant women affected by P. falciparum are also susceptible to the complications of pregnancy due to placental parasite sequestration. Treatment of malaria in pregnancy must be managed with an ID specialist and should be treated with IV antimalarial therapy. The test of choice is Giesma-stained thick and thin blood smears. Thick films sensitively detect parasites whereas thin films allow species identification and calculation of parasitaemia to guide treatment. Studies have shown that for P falciparum, the most effective treatment is artesunate which is more effective than quinine without the risk of cinchonism. Numerous studies such as the AQUAMAT study in The Lancet showing that quinine should no longer be the established treatment of choice.
A 22 year old man presented with a two week history of fever and drenching night sweats. He had experienced severe itching during this time. Examination was normal except for swollen supraclavicular lymph nodes. CXR showed a mediastinal mass.
What is the most likely diagnosis? A. Post immunisation B. Malaria C. HIV infection D. Hodgkin’s lymphoma E. Appendicitis F. Systemic lupus erythematosis G. Pyelonephritis H. Sarcoidosis I. Drug reaction J. Influenza K. Glandular fever L. Gastric carcinoma M. Pneumonia N. Tuberculosis
D. Hodgkin’s lymphoma
This is a case of lymphoma. Reed-Sternberg cells are binucleate cells characteristically seen in Hodgkin’s lymphoma. Hodgkin’s is localised to a single group of nodes (normally the cervical and/or supraclavicular) and extranodal involvement is rare. Mediastinal involvement is common. Spread is contiguous and B symptoms may be present such as a low grade fever, weight loss and night sweats. Pruritis may be found in approximately 10% of cases but has no prognostic significance. 50% of cases is associated with EBV infection and distribution is bimodal with peaks in young and old. There is classically pain in lymph nodes on alcohol consumption.
A 35 year old pregnant woman developed a temperature with chills and increased urinary frequency. She is tender in the right loin and has vomited. Dipstick urinalysis is positive for leukocytes, nitrites and blood.
What is the most likely diagnosis? A. Post immunisation B. Malaria C. HIV infection D. Hodgkin’s lymphoma E. Appendicitis F. Systemic lupus erythematosis G. Pyelonephritis H. Sarcoidosis I. Drug reaction J. Influenza K. Glandular fever L. Gastric carcinoma M. Pneumonia N. Tuberculosis
G. Pyelonephritis
Acute onset fever with chills, flank pain, vomiting and positive urine dipstick all point to the diagnosis of acute pyelonephritis. Urinalysis is highly sensitive but not very specific. Pregnancy is a risk factor for complicated disease as the enlarging uterus compresses the ureters and hormonal changes increase the likelihood of obstructive uropathy. In uncomplicated pyelonephritis, the most common cause is E. coli and gram stain will typically reveal gram negative rods, either E. coli, Proteus or Klebsiella. Gram positive cocci that could be implicated include enterococci and staphylococci. Older patients can often also present non-specifically. Treatment should start before culture results are received to prevent the patient from deteriorating, with empirical antibiotics.
A 50 year old woman presented with a temperature and aching joints, 2 days prior to her departure on holiday to Egypt. On examination she had a tender swelling on her left upper arm.
What is the most likely diagnosis? A. Post immunisation B. Malaria C. HIV infection D. Hodgkin’s lymphoma E. Appendicitis F. Systemic lupus erythematosis G. Pyelonephritis H. Sarcoidosis I. Drug reaction J. Influenza K. Glandular fever L. Gastric carcinoma M. Pneumonia N. Tuberculosis
A. Post immunisation
Travel to Egypt may necessitate vaccinations for diseases such as typhoid and HAV. This is a side-effect of the vaccine and the history here points towards this. Vaccine side effects tend to be uncommon although this depends on the specific vaccine given. For example, some 10-30% of people will experience mild side effects such as muscle pain and headache after being given the yellow fever vaccine.
A 35 year old woman complains of weight loss, tremors, palpitations, a large appetite and diarrhoea. Examination revealed tachycardia over 120 beats/min and exophthalmos. She appeared anxious.
What is the most likely diagnosis? A. Infected dermoid cyst B. Anaplastic carcinoma C. Anxiety state D. Myxoedema E. Toxic multinodular goitre F. Thyroglossal cyst G. Pituitary failure H. Viral thyroiditis I. Grave’s disease J. Papillary thyroid carcinoma K. Medullary thyroid carcinoma L. Carotid body tumour
I. Grave’s disease
This patient is obviously hyperthyroid with the weight loss despite an increased appetite, tremor, palpitations, tachycardia and diarrhoea. This patient has Graves’ disease, which is the most common cause of hyperthyroidism in countries with an adequate iodine intake. Peripheral manifestations, such as in this case exopthalmos (and pretibial myxoedema, hyperthyroid acropachy), do not occur with other causes of hyperthyroidism. Where there is pretibial myxoedema, there is almost always opthalmopathy too. Treatment aims to normalise thyroid function and is achieved by radioactive iodine, antithyroid medications or with surgery. They are all effective and relatively safe options. Symptomatic therapy is given with beta blockers such as propranolol.
50 yr old woman complains of weight loss, tremor of hands, palpitations, large appetite, diarrhoea and an irregular goitre in the neck. The patient is tachycardiac and eye examination revealed no exopthalmos or lid lag.
What is the most likely diagnosis? A. Infected dermoid cyst B. Anaplastic carcinoma C. Anxiety state D. Myxoedema E. Toxic multinodular goitre F. Thyroglossal cyst G. Pituitary failure H. Viral thyroiditis I. Grave’s disease J. Papillary thyroid carcinoma K. Medullary thyroid carcinoma L. Carotid body tumour
E. Toxic multinodular goitre
This patient is again obviously hyperthyroid with the symptoms she is presenting with but does not have the peripheral stigmata of Graves’. This is a case of toxic multinodular goitre characterised by the irregular goitre, rather than the smooth goitre of Graves’. There may be substernal extension. It is most common in older patients and is associated with head and neck irradiation and iodine deficiency. TSH is the initial screening test and if supressed, T4/T3 levels are measured. A thyroid scan and uptake will show multiple hot and cold areas consistent with areas of autonomy and areas of suppression. Definitive treatment is commonly given in the form of radioactive iodine.
24 yr old woman complains of weight loss, tremor of hands, palpitations and loss of appetite. Examination revealed tachycardia and she appeared nervous. Investigation revealed TSH 1.0 (range 0.3-4.5) and ESR within normal limits.
What is the most likely diagnosis? A. Infected dermoid cyst B. Anaplastic carcinoma C. Anxiety state D. Myxoedema E. Toxic multinodular goitre F. Thyroglossal cyst G. Pituitary failure H. Viral thyroiditis I. Grave’s disease J. Papillary thyroid carcinoma K. Medullary thyroid carcinoma L. Carotid body tumour
C. Anxiety state
TSH is normal so there is no thyroid disorder here despite the patient’s symptoms and the patient also appears nervous. Anxiety would explain the patient’s symptoms here and weight loss may be incorporated as part of an anxiety disorder.
A 24 year old woman complains of weight loss, tremor of hands, palpitations and pain in neck radiating to the left ear and tiredness. Examination revealed tender swelling in neck, tachycardia of 120 beats/min and fever of 38oC. TSH <0.05 (range 0.3-4.5) and ESR 60mm/hr.
What is the most likely diagnosis? A. Infected dermoid cyst B. Anaplastic carcinoma C. Anxiety state D. Myxoedema E. Toxic multinodular goitre F. Thyroglossal cyst G. Pituitary failure H. Viral thyroiditis I. Grave’s disease J. Papillary thyroid carcinoma K. Medullary thyroid carcinoma L. Carotid body tumour
H. Viral thyroiditis
This woman has de Quervain’s (viral) thyroiditis (which can also be called subacute granulomatous thyroiditis) which is inflammation of the thyroid characterised by a triphasic course where there is transient thyrotoxicosis followed by a hypothyroid phase before a return to euthyroidism. The thyrotoxic phase (symptoms of hyperthyroidism may are present here) is characterised by pain and tenderness of the thyroid, which tends to be larger, firm and tender to touch. This woman also has a fever and neck pain. This is a self-limiting condition and no specific treatment is needed though NSAIDs and beta blockers can be used for symptomatic relief. Roughly 30-40% describe a prior viral infection.
An 18 year old woman complains of a swelling in the neck. Examination revealed the swelling is painless, non-tender. There is no weight loss, no tremors of hands or palpitations. Swelling moves up and down with protrusion and intrusion of tongue respectively. Investigation revealed normal TSH and ESR.
What is the most likely diagnosis? A. Infected dermoid cyst B. Anaplastic carcinoma C. Anxiety state D. Myxoedema E. Toxic multinodular goitre F. Thyroglossal cyst G. Pituitary failure H. Viral thyroiditis I. Grave’s disease J. Papillary thyroid carcinoma K. Medullary thyroid carcinoma L. Carotid body tumour
F. Thyroglossal cyst
This midline neck swelling moves up on tongue protrusion (and swallowing) making this a thyroglossal cyst. It is a cyst that forms from a remnant thyroglossal duct and can hence develop anywhere along the length of this embryological duct, which is a midline structure between the foramen caecum at the back of the tongue and the thyroid gland.
A 50 yr old man complains of a lump in neck on the left side. Examination revealed painless, non tender lump on the left side of the neck which moves on swallowing. Investigation revealed normal TSH and ESR. Fine needle aspiration revealed ‘Orphan Annie’ eyes and psammoma bodies.
What is the most likely diagnosis? A. Infected dermoid cyst B. Anaplastic carcinoma C. Anxiety state D. Myxoedema E. Toxic multinodular goitre F. Thyroglossal cyst G. Pituitary failure H. Viral thyroiditis I. Grave’s disease J. Papillary thyroid carcinoma K. Medullary thyroid carcinoma L. Carotid body tumour
J. Papillary thyroid carcinoma
TSH is normal and the examination findings point to a carcinoma, which most commonly presents like this – an asymptomatic nodule. If TSH were suppressed then this would suggest hyperthyroidism, or a hot nodule, in which case the incidence of cancer is very low. If TSH, like this case, is normal, then a FNA is indicated and cytology may then help to tell cancer type. Histology cannot distinguish follicular adenomas and carcinomas but is used for diagnosis of follicular carcinoma (when combined with other features) instead of cytology. There are 5 types of thyroid cancer: papillary (most common), follicular, medullary (about a quarter are familial e.g. MEN), anaplastic (worst prognosis) and lymphoma. Orphan Annie eyes and psammoma bodies are seen in papillary cancer. Average 10 year survival is >90%. This is a hard question.
A 20 year old male with generalised oedema. He has noticed ‘frothy’ urine recently. Liver function tests show plasma albumin of 15g/l.
What is the most appropriate investigation? A. 24hr urinary protein B. Lymph node biopsy C. Venous duplex scan D. Echocardiogram E. Creatinine clearance F. Liver function tests G. Drug history H. Full blood count I. Urea & electrolytes J. Polysomnography K. Pelvic ultrasound L. V/Q scan M. Blood film examination
A. 24hr urinary protein
This man has nephrotic syndrome, which is defined by the presence of proteinuria (>3.5g/24 hours), hypoalbuminaemia (<30g/L) oedema, and hyperlipidaemia. You should note that this is different to nephritic syndrome which is typically defined by acute kidney injury, hypertension and an active urinary sediment (RBCs and RBC casts). This is a constellation of several symptoms which can have many causes. The likely cause differs depending on how old the patient is and whether the patient has co-morbidities like diabetes. The most common cause in children is minimal change disease, whereas the most common cause in young adults is focal segmental glomerulosclerosis. Membranous nephropathy is most common in older people and you should suspect diabetic nephropathy as a cause in those with long standing DM. Note also that those who present with proteinuria without the other components needed to define nephrotic syndrome are described as having ‘nephrotic range proteinuria’.
In terms of physiology, the glomerular filtration barrier is disrupted by disease leading to glomerular proteinuria (the scale of which is linked to how severe the condition is and renal function). The loss of albumin leads to hypoalbuminaemia. The liver then tries to compensate for the protein loss by increases albumin synthesis, and this also increases synthesis of LDL, vLDL and lipoprotein A, leading to lipid abnormalities. The patient is also in a hypercoagulable state due to the loss of coagulatiion inhibitors in urine and increased synthesis of clotting factors in the liver. A 24-hour urine protein collection would establish proteinuria and whether it is in the nephrotic range. If tis is impractical then the protein/creatinine ratio can be obtained.
A 60 year old woman complained of a painful swollen left leg on arrival at Heathrow airport. On examination her leg is tender with shiny skin. She is on the oral contraceptive pill.
What is the most appropriate investigation? A. 24hr urinary protein B. Lymph node biopsy C. Venous duplex scan D. Echocardiogram E. Creatinine clearance F. Liver function tests G. Drug history H. Full blood count I. Urea & electrolytes J. Polysomnography K. Pelvic ultrasound L. V/Q scan M. Blood film examination
C. Venous duplex scan
This is a DVT as I am sure you have already figured out.The recent presumably long-haul air travel is a strong risk factor though the exact mechanism of this risk is currently unclear, but may involve pressure changes, dehydration and mobility. This woman also has another risk factor with the use of oestrogen-containing oral contraceptives, although this is a small individual risk. The risk here is greatest in the first year of use. Other strong risk factors include recent surgery, especially orthopaedic surgery, active malignancy, pregnancy, obesity and coagulopathies such as factor V Leiden. A Wells score is determined in all patients with a suspected DVT with the condition being likely if the score is 2 or more. If the Wells score is <2 then a D-dimer level is indicated. A compression ultrasound of the proximal deep venous system is the first line investigation in those where there is a high clinical suspicion. However, Doppler venous flow studies can be used when all other tests are unavailable although this test has a low sensitivity and requires a trained technician. A low flow in the veins is indicative of a DVT.
A 45 year old publican developed bilateral limb oedema and noticed that his trousers were becoming tight. On examination he had gynaecomastia and Duputyren’s contractures
What is the most appropriate investigation? A. 24hr urinary protein B. Lymph node biopsy C. Venous duplex scan D. Echocardiogram E. Creatinine clearance G. Drug history H. Full blood count I. Urea & electrolytes J. Polysomnography K. Pelvic ultrasound L. V/Q scan M. Blood film examination
F. Liver function tests
This is a sign of decompensation of hepatic cirrhosis which has caused peripheral oedema due to hypoalbuminaemia. The gynaecomastia is a sign of chronic liver disease (tender and firm enlarged breast bud) as well as the Dupuytren’s contracture of alcoholic liver disease. Cirrhosis is the end-stage of chronic liver disease, in this case due to alcohol. Cirrhosis results in hepatic insufficiency and portal hypertension. Other complications include ascites, variceal bleeds, jaundice, hepatic encephalopathy, hepatorenal syndrome and the development of HCC. The only curative option after decompensation is a timely referral for liver transplantation. Management is aimed at treating the underlying liver disease – this man needs to stop drinking.
A 60 year old female with uncontrolled hypertension, whose medication has been recently changed by he GP presents with bilateral ankle swelling. Clinical examination of her chest and cardiovascular system were normal.
What is the most appropriate investigation? A. 24hr urinary protein B. Lymph node biopsy C. Venous duplex scan D. Echocardiogram E. Creatinine clearance F. Liver function tests H. Full blood count I. Urea & electrolytes J. Polysomnography K. Pelvic ultrasound L. V/Q scan M. Blood film examination
G. Drug history
A new change in medication here has led to peripheral oedema. The examination is normal which would lead you away from suspecting causes such as heart failure (plus the patient does not seem to be complaining of any other symptoms such as SOB). The JVP is normal as well. These agents which can cause bilateral oedema include CCBs, especially the dihydropyridines and agents which cause vasodilation such as minoxidil or diazoxide. Drugs which act to block renal prostaglandin synthesis such as NSAIDs can also lead to renal salt and water retention which would also contribute to peripheral oedema. There are other medications associated with peripheral oedema such as antidepressants, oestrogens and steroids. The clinical suspicion here should very much lie towards the new medication, which has coincided with the onset of oedema. This diagnosis would be supported by improvement in oedema with the withdrawal of the offending drug.
A 70 year old woman with swelling of her right leg. She gives a history of recent weight loss and lower abdominal discomfort. Examination shows pitting oedema of the entire leg and a possible right sided pelvic mass.
What is the most appropriate investigation? A. 24hr urinary protein B. Lymph node biopsy C. Venous duplex scan D. Echocardiogram E. Creatinine clearance F. Liver function tests G. Drug history H. Full blood count I. Urea & electrolytes J. Polysomnography K. Pelvic ultrasound L. V/Q scan M. Blood film examination
K. Pelvic ultrasound
There is a possible mass in the pelvis which is most likely malignant given the history. This needs to be investigated by pelvic ultrasound. This is likely secondary lymphoedema due to malignancy with the weight loss and abdominal discomfort. This is initially pitting. Stemmer’s sign may be useful, which is the inability to pinch and lift a fold of skin at the base of the second toe.
A 60 year old man who is a heavy smoker presents with a longstanding history of cough and breathlessness. He has recently noticed increasing difficulty in walking due to leg swelling. On examination he is centrally cyanosed and has bilateral wheezes on chest auscultation.
What is the most appropriate investigation? A. 24hr urinary protein B. Lymph node biopsy C. Venous duplex scan D. Echocardiogram E. Creatinine clearance F. Liver function tests G. Drug history H. Full blood count I. Urea & electrolytes J. Polysomnography K. Pelvic ultrasound L. V/Q scan M. Blood film examination
D. Echocardiogram
This is cor pulmonale in a patient with a history of COPD. There is peripheral oedema and wheezes on chest examination. Cor pulmonale is right heart failure secondary in this case to long standing COPD, caused by chronic hypoxia and pulmonary vascular vasoconstriction secondary to this, giving pulmonary hypertension and right sided heart failure. Signs aside from lower extremity oedema also include hepatomegaly, a loud P2 and engorged neck veins. The first test to order is an echocardiogram which will slow elevated pulmonary arterial pressure. Further tests may include spirometry and CT chest.
A 30 year old female smoker has atrophic waxy yellowish plaques on her shins with surrounding erythema.
What is the most likely diagnosis? A. Diabetes B. Scleroderma C. Ulcerative colitis D. Hyperthyroidism E. Dermatomyositis F. Amyloidosis G. Reactive arthritis H. Rheumatoid arthritis I. Hypothyroidism J. Coeliac disease
A. Diabetes
This is necrobiosis lipoidica which is a necrotising condition usually occuring in those with diabetes (and can be called necrobiosis lipodica diabeticorum). Most people with the condition are diabetics but only a small proportion of diabetics have the condition. It tends to appear on the shins, often bilaterally, although it can occur on other sites. They are often asymptomatic (due to associated neuropathy) but may be painful and ulcerate if injured. It tends to have the appearance of an erthematous area of hardened and raised skin (initially a reddish brown), progressing to a yellowish tint. Try and have a look at some photos online. These lesions may display the Koebner phenomenon.
A 35 year old woman has tight shiny skin on her hands & around her mouth. There are also telangiectasia on her face. She has had Raynaud’s phenomenon for the past 5 years and a history of GORD.
What is the most likely diagnosis? A. Diabetes B. Scleroderma C. Ulcerative colitis D. Hyperthyroidism E. Dermatomyositis F. Amyloidosis G. Reactive arthritis H. Rheumatoid arthritis I. Hypothyroidism J. Coeliac disease
B. Scleroderma
This is scleroderma, or systemic sclerosis. It is a multi-system disease with initially non-specific symptoms such as tiredness, hand swelling, musculoskeletal symptoms and Raynaud’s phenomenon. The cause is currently unknown. There are two main types – the limited cutaneous type and the diffuse cutaneous tupe. The limited cutaneous form tends to have less severe internal organ involvement and a better prognosis. Telangiectasia is commonly found, most often on the fingers, palms, face and mucous membranes. They can also be found in the stomach, where they are referred to as ‘watermelon stomach’. Characteristic findings of this condition also include sclerodactyly and thickening of the skin proximal to the MCP jpints. GORD, digital ulcers and lung involvement may also be found. Clinical course depends on the extent of both vascular and fibrotic complications. Vascular involvement includes Raynaud’s, ischaemic digital ulcers and hypertensive crisis as well as pulmonary arterial hypertension. Fibrosis can affect lungs, heart and GIT.
An 18 year old with a history of episodes of diarrhoea, in between which she suffers no symptoms, has painful pustular lesions which are ulcerating.
What is the most likely diagnosis? A. Diabetes B. Scleroderma C. Ulcerative colitis D. Hyperthyroidism E. Dermatomyositis F. Amyloidosis G. Reactive arthritis H. Rheumatoid arthritis I. Hypothyroidism J. Coeliac disease
C. Ulcerative colitis
This is pyoderma gangrenosum, which presents with multiple lesions, most commonly affecting the lower extremity and is linked to UC although it can be seen less commonly in Crohn’s, and is also seen in conditions such as RA and the myeloid dyscrasias. However, the diarrhoea here in the history makes this UC. These lesions start as tender papules or vesicles which develop into painful ulcers with a dusky purple edge and surrounding induration and erythema. The base may contain granulation tissue and lesions heal with atrophic scars. This patient will need a colonoscopy to investigate for UC. Skin culture may also reveal infection, which can occur. Physical trauma such as debridement is contraindicated as it can cause severe exacerbations.
A 28 year old woman has indurated plaques of reddish-yellow thickened skin over both shins. She also has areas of depigmentation on her hands & face.
What is the most likely diagnosis? A. Diabetes B. Scleroderma C. Ulcerative colitis D. Hyperthyroidism E. Dermatomyositis F. Amyloidosis G. Reactive arthritis H. Rheumatoid arthritis I. Hypothyroidism J. Coeliac disease
D. Hyperthyroidism
This is pretibial myxoedema and vitiligo in a woman with Graves’ disease. Graves’ disease is the most common cause of hyperthyroidism. Peripheral manifestations such as ophthalmopathy, pretibial myxoedema and hyperthyroid acropachy do not occur with other causes of hyperthyroidism. Pretibial myxoedema is almost always associated with ophthalmopathy. Treatment aims to normalise thyroid function and is achieved by radioactive iodine, antithyroid medications or with surgery. They are all effective and relatively safe options. Symptomatic therapy is given with beta blockers such as propranolol.
A 65 year old man with painful shoulders & hips has purple swollen eyelids & red scaly papules over the knuckles.
What is the most likely diagnosis? A. Diabetes B. Scleroderma C. Ulcerative colitis D. Hyperthyroidism E. Dermatomyositis F. Amyloidosis G. Reactive arthritis H. Rheumatoid arthritis I. Hypothyroidism J. Coeliac disease
E. Dermatomyositis
Dermatomyositis is an idiopathic myopathy which is characterised by hallmark cutaneous lesions such as a heliotrope rash and Gottron’s papules. Gottron’s papules are seen here and this is a pathognomic sign characterised by violet to dusty-red papules and plaques over the dorsal surface of the knuckles, wrists, elbows, knees and malleoli. The surface may be slightly scaling and telangiectasia may develop within the lesions. The periorbital violet rash in a symmetrical involvement is also a very characteristic sign and there may also be associated periorbital oedema. Skin manifestations are treated with topical antipruritics and topical corticosteroids. Photoprotection is advised for all patients – the rash often develops in sun exposed areas and some patients also report photosensitivity. UV radiation is a strong risk factor for this disease. Some patients only have cutaneous involvement whereas others may have associated lung disease, heart involvement, oesophageal involvement of underlying malignancy. Muscle disease is initially treated with high dose prednisolone followed by immunosuppresants and IV immunoglobulins in refractory cases.
A 21 year old Jamaican gentleman was referred to A&E by his GP, who on investigation following a chest infection found an incidental finding of a raised bilirubin of 120micromols/L. His LFTs are within normal ranges. O/E he was found to be jaundiced but he said that he had “always been that way”.
What is the most likely diagnosis? A. Gilbert's syndrome B. Hepatitis C C. Primary biliary cirrhosis D. Hepatocellular carcinoma E. Hepatorenal syndrome F. Hepatitis E G. Hepatitis D H. Hepatitis A I. Hepatitis B J. Portal hypertension K. Gallstones L. Pyogenic abscess M. Alcoholic liver disease N. GORD
A. Gilbert’s syndrome
Gilbert’s occurs in an asymptomatic patient, often as an incidental finding or mild jaundice occuring in adolescence/young adult age. There is elevated unconjugated BR with other liver tests being normal. The blood smear is also normal with normal reticulocyte count, and normal Hb indicating that this is not due to haemolysis. It is a common syndrome and is not really a disease, more a physiological variant. No treatment is needed and this condition is due to decreased UDPGT activity leading to decreased conjugation of unconjugated bilirubin, leading to elevated levels.
Mrs C is a 55 year old mother of two. Who has been feeling increasingly tired over the past few months. Investigations showed an ALT of 497. Her past medical history includes type 2 diabetes and arthritis. Her only other hospital admission was 15 years ago was when her youngest girl was born when she had major complications and needed several blood transfusions. After a few more tests the doctor prescribes peginterferon and ribavirin.
What is the most likely diagnosis? A. Gilbert's syndrome B. Hepatitis C C. Primary biliary cirrhosis D. Hepatocellular carcinoma E. Hepatorenal syndrome F. Hepatitis E G. Hepatitis D H. Hepatitis A I. Hepatitis B J. Portal hypertension K. Gallstones L. Pyogenic abscess M. Alcoholic liver disease N. GORD
I. Hepatitis C
The most common route which HCV is transmitted is through illicit IVDU. Following the acute exposure, most patients (55-85%) will go on to develop chronic hepatitis C. Most infections are asymptomatic. Treatment involves, usually, pegylated interferon and ribavirin with the aim of getting rid of viraemia. Triple therapy is used for the most common genotype 1, and this consists of the above two drugs as well as a HCV protease inhibitor. Long term complications of infection include cirrhosis or HCC.
The first case is one of transfusion-associated HCV, which occured before donor screening was put in place. This is currently an incredibly rare occurence in western countries. The serum transaminases, especially ALT are the easiest measure of disease activity (but are neither very sensitive nor very specific). The association between ALT level and the degree of chronic liver disease is however weak and ALT level does not reflect the progression to cirrhosis. Pegylated interferon itself can also cause mild ALT elevation. This is not HBV because of the last line which gives away the treatment, consistent with HCV. If this information was not given, this would still be more likely to be HCV than HBV as HCV has a greater chance of establishing chronic infection. However, there may be no differentiating signs or symptoms between the two. HBV surface antigen test would be positive if it was HBV and HCV antibody test negative.
The second case has two risk factors and has presented with classic signs of liver disease with ascites as a sign of decompensation. Tattoos are associated with HCV infection as is homosexuality. It is really important to note that HCV is not sexually transmitted. This may sound like a paradox… but HCV is trasmitted by blood, not sex. The reason homosexual intercourse is a risk factor for HCV and HCV is increasing in incidence among homosexual men in London is likely due to traumatic anal intercourse. Hence, the virus is still transmitted by blood. This is what the majority of studies seem to suggest, although there is still a bit of contention here, as there is with whether HIV can be transmitted through oral sex. HBV markers are negative here. Surface antigen appears 2-10 weeks after exposure and is usually detectable after 4-6 months with persistence indicating chronic infection. E antigen is a soluble viral protein sound in serum in the early acute phase of infection and usually disappears soon after or at the peak in ALT levels. Persistence again suggest chronicity.
Mr H is a 46 year old homosexual tattoo artist who presents to his GP because he has noticed worsening abdominal swelling and dark stool. On examination you notice spider naevi on the upper chest and gynaecomastia. Initial blood tests demonstrate negative HBsAg and HBeAg.
What is the most likely diagnosis? A. Gilbert's syndrome B. Hepatitis C C. Primary biliary cirrhosis D. Hepatocellular carcinoma E. Hepatorenal syndrome F. Hepatitis E G. Hepatitis D H. Hepatitis A I. Hepatitis B J. Portal hypertension K. Gallstones L. Pyogenic abscess M. Alcoholic liver disease N. GORD
B. Hepatitis C
The most common route which HCV is transmitted is through illicit IVDU. Following the acute exposure, most patients (55-85%) will go on to develop chronic hepatitis C. Most infections are asymptomatic. Treatment involves, usually, pegylated interferon and ribavirin with the aim of getting rid of viraemia. Triple therapy is used for the most common genotype 1, and this consists of the above two drugs as well as a HCV protease inhibitor. Long term complications of infection include cirrhosis or HCC.
The first case is one of transfusion-associated HCV, which occured before donor screening was put in place. This is currently an incredibly rare occurence in western countries. The serum transaminases, especially ALT are the easiest measure of disease activity (but are neither very sensitive nor very specific). The association between ALT level and the degree of chronic liver disease is however weak and ALT level does not reflect the progression to cirrhosis. Pegylated interferon itself can also cause mild ALT elevation. This is not HBV because of the last line which gives away the treatment, consistent with HCV. If this information was not given, this would still be more likely to be HCV than HBV as HCV has a greater chance of establishing chronic infection. However, there may be no differentiating signs or symptoms between the two. HBV surface antigen test would be positive if it was HBV and HCV antibody test negative.
The second case has two risk factors and has presented with classic signs of liver disease with ascites as a sign of decompensation. Tattoos are associated with HCV infection as is homosexuality. It is really important to note that HCV is not sexually transmitted. This may sound like a paradox… but HCV is trasmitted by blood, not sex. The reason homosexual intercourse is a risk factor for HCV and HCV is increasing in incidence among homosexual men in London is likely due to traumatic anal intercourse. Hence, the virus is still transmitted by blood. This is what the majority of studies seem to suggest, although there is still a bit of contention here, as there is with whether HIV can be transmitted through oral sex. HBV markers are negative here. Surface antigen appears 2-10 weeks after exposure and is usually detectable after 4-6 months with persistence indicating chronic infection. E antigen is a soluble viral protein sound in serum in the early acute phase of infection and usually disappears soon after or at the peak in ALT levels. Persistence again suggest chronicity.
Miss K is a 45 year old retired kitchen lady who presents to A+E with abdominal discomfort which has been getting worse over the past few weeks. She says she feels tired, bloated and has lost her appetite. Findings show antimitochrondrial antibodies are present and serum alkaline phosphatase is raised.
What is the most likely diagnosis? A. Gilbert's syndrome B. Hepatitis C C. Primary biliary cirrhosis D. Hepatocellular carcinoma E. Hepatorenal syndrome F. Hepatitis E G. Hepatitis D H. Hepatitis A I. Hepatitis B J. Portal hypertension K. Gallstones L. Pyogenic abscess M. Alcoholic liver disease N. GORD
C. Primary biliary cirrhosis
Primary biliary cirrhosis (PBC) is a chronic condition where the intrahepatic small bile ducts are progressively damaged (and eventually lost) occuring on a background of portal tract inflammation. Fibrosis develops, ultimately leading to cirrhosis (which is defined as fibrosis with nodular regeneration). It is widely believed to be autoimmune in aetiology as almost all patients have antimitochrondrial antibodies. This is the give away point in this question which is otherwise a non-specific presentation. Elevated ALP suggests cholestasis.
Mr S is a 70 year old retired semi professional footballer. He presents with pain in the right hypochondrium which does not radiate. On further questioning he says he has noticed a 5kg weight loss in the past couple of months and he says he has completely lost his appetite. He drinks an average of 1-2 units of alcohol per week and does not smoke. O/E gross ascites is present and obs chart shows he has been pyrexial since admission.
What is the most likely diagnosis? A. Gilbert's syndrome B. Hepatitis C C. Primary biliary cirrhosis D. Hepatocellular carcinoma E. Hepatorenal syndrome F. Hepatitis E G. Hepatitis D H. Hepatitis A I. Hepatitis B J. Portal hypertension K. Gallstones L. Pyogenic abscess M. Alcoholic liver disease N. GORD
D. Hepatocellular carcinoma
HCC usually arises in patients with cirrhosis, such as this patient, who has evidence of alcohol use and decompensation with ascites. The weight loss and the site of pain are also indicative. The RUQ pain is due to hepatomegaly, inflammation and stretching of the liver capsule. The ascites is associated with cirrhosis. A liver mass should be present on abdominal imaging and elevated AFP would be a corroborative finding. Treatment depends on the stage and prognosis. If a patient is previously known to have cirrhosis or are at a high risk then they should really be screened for a hepatoma by obtaining serum AFP and USS of the liver every 6 months. Treatment options would include: resection, transplant, chemo-embolisation and percutaneous ablation.
A 47 year old presents with a sharp central chest pain, worse on lying down, with a two week history of fever and malaise.
What is the most likely diagnosis? A. GORD B. Pericardial effusion C. Stable angina D. Pulmonary embolism E. Unstable angina F. Pleural effusion G. Myocardial infarction H. Pericarditis I. Pneumonia J. Pneumothorax K. Emphysema L.Peptic ulcer M. Fractured rib N. Asthma O. Ventricular fibrillation P. Aortic dissection
H. Pericarditis
This patient has presented with pericarditis. Symptoms include a sharp and severe chest pain retrosternally which is worse on inspiration and when supine, relieved by sitting forwards. The classical finding on examination is a friction rub which is said to sound like ‘walking on snow’. There may be diffuse ST elevations on ECG, an effusion on echocardiography and blood results suggesting inflammation. Complications include tamponade and constrictive pericarditis. Prior viral infection is a risk factor with the most common pericardial infection being viral. Bacterial purulent pericarditis also occurs. The inflammation is due either to direct viral attack or immune mediated damage. Other risk factors include male gender, post-MI (both ‘early’ and Dressler’s), post-pericardiotomy syndrome, neoplasm from local tumour invasion, uraemia and autoimmune conditions such as RA and SLE.
A 25 year old presents with chest pain, worse on inspiration and associated with a productive cough.
What is the most likely diagnosis? A. GORD B. Pericardial effusion C. Stable angina D. Pulmonary embolism E. Unstable angina F. Pleural effusion G. Myocardial infarction H. Pericarditis J. Pneumothorax K. Emphysema L.Peptic ulcer M. Fractured rib N. Asthma O. Ventricular fibrillation P. Aortic dissection
I. Pneumonia
This case of pleuritic chest pain and a productive cough is classic pneumonia. The most specific and sensitive test is a CXR (PA and lateral) and initial treatment of a CAP is empirical with antibiotics. Often diagnosis is made solely on history and examination findings. Management is guided by the patient’s CURB-65 score.
A 54 year old with severe sudden onset pain mainly felt between the scapulae.
What is the most likely diagnosis? A. GORD B. Pericardial effusion C. Stable angina D. Pulmonary embolism E. Unstable angina F. Pleural effusion G. Myocardial infarction H. Pericarditis I. Pneumonia J. Pneumothorax K. Emphysema L.Peptic ulcer M. Fractured rib N. Asthma O. Ventricular fibrillation P. Aortic dissection
P. Aortic dissection
The tearing chest pain suggests aortic dissection. There may also be interscapular pain with dissection of the descending aorta. Dissecting aneurysms are either type A, which involves the ascending aorta, or type B. Type A dissections require urgent surgery whereas type B can be managed medically if it is not complicated by end organ ischaemia. BP differential between the 2 arms is a hallmark feature. Pulse differences may also be present in the lower limbs. There may also be the diastolic murmur of AR in proximal dissections. A CT scan is indicated as soon as a diagnosis of aortic dissection is suspected and should be from the chest to the pelvis to see the full extent of the dissecting aneurysm. What you will see is the intimal flap. MRI is more sensitive and specific but is more difficult to obtain acutely.
Progressive retrosternal pain in a 45 year old patient known to have a hiatus hernia.
What is the most likely diagnosis? A. GORD B. Pericardial effusion C. Stable angina D. Pulmonary embolism E. Unstable angina F. Pleural effusion G. Myocardial infarction H. Pericarditis I. Pneumonia J. Pneumothorax K. Emphysema L.Peptic ulcer M. Fractured rib N. Asthma O. Ventricular fibrillation P. Aortic dissection
A. GORD
This patient has GORD characterised by heartburn and regurgitation of acid. It is more severe at night when the patient is lying flat and also when the patient is bending over. Risk factors include obesity and hiatus hernia. Diagnosis is generally clinical and can also be achieved by a diagnostic trial of a PPI. Normally an upper GI endoscopy is reserved for complications such as strictures, Barrett’s or cancer, or for atypical features. An OGD may show oesophagitis or Barrett’s (red velvety), however OGD may be normal. Manometry and pH monitoring may also be performed, but in this case, this patient will probably just have a therapeutic and diagnostic trial of a PPI instead of an OGD.
A 26 year old professional basketball player presents with central chest pain and breathlessness, with the pain worse on inspiration.
What is the most likely diagnosis? A. GORD B. Pericardial effusion C. Stable angina D. Pulmonary embolism E. Unstable angina F. Pleural effusion G. Myocardial infarction H. Pericarditis I. Pneumonia J. Pneumothorax K. Emphysema L.Peptic ulcer M. Fractured rib N. Asthma O. Ventricular fibrillation P. Aortic dissection
J. Pneumothorax
This patient has developed a primary pneumothorax. Primary pneumothoraces occur in young people without known lung conditions. The main investigation is a CXR and pneumothoraces are classified by the BTS as large (>2cm visible rim between the lung margin and the chest wall) or small (<2cm). If the patient is clinically stable, they can be observed and given oxygen – an invasive approach is not necessary and the oxygen will increase the rate of pneumothorax reabsorption. A pneumothorax, if left, is something that resolves by itself over time by reabsorption. Cavemen (and cavewomen) who got pneumothoraces all those years ago didn’t just walk around breathless until they died. If large however, percutaneous needle aspiration is required (IV cannula 2nd intercostal space, or 3rd, at the MCL). A CXR should be obtained after this procedure. If this fails, a chest drain should be inserted.
A 22 year old man walks into A&E, he was involved in a fight in a bar. As he left the bar he complained of drowsiness and double vision worst when walked down stairs.
Which nerve is most likely to have been affected? A. Cranial nerve Vb B. Cranial nerve XI C. Optic tract D. Cerebellum E. Cranial nerve IV F. Cranial nerve XII G. Cranial nerve IX H. Cranial nerve II I. Cranial nerve Va J. Cranial nerve VIII K. Cranial nerve Vc L. Cranial nerve III M. Cranial nerve I N. Cranial nerve X O. Cranial nerve VI P. Cranial nerve VII
E. Cranial nerve IV
This man is complaining of double vision. You should already be thinking III, IV or VI which are the cranial nerves responsible for eye movements. CN IV innervates the superior oblique muscle which controls depression, intorsion and adduction of the eye. This is vertical diplopia (worse when walking down the stairs) hence the cause is a lesion in CN IV. The trochlear nucleus is located in the midbrain at the level of the inferior colliculus and fasciles decussate at the medullary velum to exit at the dorsal midbrain. The nerve enters the orbit through the superior orbital fissure. Testing is done at the bedside by asking the patient to follow the examiner’s finger with the eyes, while observing eye movements and asking the patient to report any diplopia. Remember that diplopia is maximal in the direction of action of the paralysed muscle (which makes sense if you think about it) and the outer image is the image which is false, and disappears when the ipsilateral eye is covered up.
Shortly after recovering from thyroid surgery, Mrs Sugav complains of voice loss and a dry persistent cough.
Which nerve is most likely to have been affected? A. Cranial nerve Vb B. Cranial nerve XI C. Optic tract D. Cerebellum E. Cranial nerve IV F. Cranial nerve XII G. Cranial nerve IX H. Cranial nerve II I. Cranial nerve Va J. Cranial nerve VIII K. Cranial nerve Vc L. Cranial nerve III M. Cranial nerve I N. Cranial nerve X O. Cranial nerve VI P. Cranial nerve VII
N. Cranial nerve X
The vagus nerve contains both visceral efferents and afferents and has 3 main motor branches. It innervates muscles of the larynx and pharynx, with the exception of stylopharyngeus and tensor veli palatini. It also mediates sensory input from these areas as well as the external auditory canal. It also conveys visceral afferent input from the thoracic and abdominal viscera and supplies parasympathetic output to these regions as well. The nerve has been damaged with thyroid surgery and there is voice loss and a cough as a response. Signs of CN X damage include palatal droop, deviation of the uvula contralaterally and an absent gag reflex, as the efferent limb of this reflex is formed by the vagus nerve.
A patient is diagnosed with bulbar palsy. On examination, his tongue was deviated to one side.
Which nerve is most likely to have been affected? A. Cranial nerve Vb B. Cranial nerve XI C. Optic tract D. Cerebellum E. Cranial nerve IV G. Cranial nerve IX H. Cranial nerve II I. Cranial nerve Va J. Cranial nerve VIII K. Cranial nerve Vc L. Cranial nerve III M. Cranial nerve I N. Cranial nerve X O. Cranial nerve VI P. Cranial nerve VII
F. Cranial nerve XII
Bulbar palsy would give progressive symptoms such as difficulty swallowing, chewing or talking and muscle weakness. Tongue wasting, weakness and fasciculations may also be seen and an absent gag reflex may be found. The examination here shows tongue deviation in line with a CN XII lesion. The tongue deviates ipsilaterally to the lesion. There may also be tongue weakness, fasciculation at rest, flaccidity, atrophy or the inability to move the tongue rapidly from side to side or up and down. The 12th cranial nerve is purely motor in function, moving and altering the shape of the tongue by innervating the intrinsic and extrinsic tongue muscles.
A 79 year old lady was brought to A&E after a fall. On examination, when asked to close her eyes, her eyeballs rolled upwards and she was unable to close her eyes.
Which nerve is most likely to have been affected? A. Cranial nerve Vb B. Cranial nerve XI C. Optic tract D. Cerebellum E. Cranial nerve IV F. Cranial nerve XII G. Cranial nerve IX H. Cranial nerve II I. Cranial nerve Va J. Cranial nerve VIII K. Cranial nerve Vc L. Cranial nerve III M. Cranial nerve I N. Cranial nerve X O. Cranial nerve VI P. Cranial nerve VII
P. Cranial nerve VII
This is someone who cannot close their eyes. Eye closure relies on orbicularis oculi which is innervated by the facial nerve (VII). It is the most common CN mononeuropathy and the most common cause of a peripheral facial palsy is Bell’s palsy. This is why when testing CN VII you ask a patient to screw their eyes up tightly and resist you trying to open them as well as asking them to do other seemingly odd things like raising their eyebrows against resistance, showing you their teeth and pursing their lips. The facial nerve is both motor and sensory (via the nervus intermedius).The nerve exits the cranium through the stylomastoid foramen and enters the parotid, where it splits into 5 branches – temporal, zygomatic, buccal, mandibular and cervical.
A middle aged lady brought into A&E after a RTA, on examination was found unable to discriminate between smells. Her nasal passages were found to be clear.
Which nerve is most likely to have been affected? A. Cranial nerve Vb B. Cranial nerve XI C. Optic tract D. Cerebellum E. Cranial nerve IV F. Cranial nerve XII G. Cranial nerve IX H. Cranial nerve II I. Cranial nerve Va J. Cranial nerve VIII K. Cranial nerve Vc L. Cranial nerve III M. Cranial nerve I N. Cranial nerve X O. Cranial nerve VI P. Cranial nerve VII
M. Cranial nerve I
This woman has dysnosmia (distortions of smell) caused by dysfunction of cranial nerve I, the olfactory nerve. Anosmia refers to the complete loss of smell and hyposmia refers to the partial loss of smell. This is a clinical diagnosis using odour identification tests where patients are asked to identify pre-defined smells. Specialist centres can do olfactory evoked potentials. The odorants are transported to bipolar cells located in the olfactory neuroepithelium and these cells synapse with olfactory bulb glomeruli. The paired olfactory bulbs are located at the base of the frontal lobe overlying the cribriform plate, and during trauma like a road traffic accident, the connections at the cribriform plate can be broken.
A 40 year old gentleman comes to A&E, with severe pain in his right loin, radiating to the groin. His PMH includes several episodes of UTIs.
What is the most likely diagnosis? A. Rheumatoid arthritis B. Nephrotic syndrome C. Prostate carcinoma D. Polycystic kidneys E. Bladder stones F. Bladder carcinoma G. Ureteric stones H. Glomerulonephritis I. Appendicitis J. SLE K. UTI L. Urinary tract obstruction M. Renal cell carcinoma N. Gout O. Acute tubular necrosis
G. Ureteric stones
This patient has renal colic which classically presents with severe flank pain radiating to the groin. Infection may complicate renal calculi. Microscopic haematuria is present in up to 90% of cases. Up to 85% of stones are visible on a plain KUB although urate stones are radiolucent. If the stone is radio-opaque, calcification will be seen within the urinary tract. In pregnancy though, a renal USS is first line. The IVP has now been replaced by the CT scan which is the new diagnostic standard. A non-contrast helical (or spiral) CT is preferred due to high sensitivity and specificity and acurately determines presence, site and size of stones. Stones are analysed after they are extracted or when they are expelled to check their composition.
A 70 year old gentleman presents with increased urine frequency and nocturia up to 5 times/night. On direct questioning, it is also noted that he also has urinary hesitancy and terminal dribbling, and that he thinks he has recently lost weight.
What is the most likely diagnosis? A. Rheumatoid arthritis B. Nephrotic syndrome D. Polycystic kidneys E. Bladder stones F. Bladder carcinoma G. Ureteric stones H. Glomerulonephritis I. Appendicitis J. SLE K. UTI L. Urinary tract obstruction M. Renal cell carcinoma N. Gout O. Acute tubular necrosis
C. Prostate carcinoma
This patient has symptoms of urinary tract outfow obstruction – namely nocturia, frequency and hesitancy. Haematuria and dysuria may be complaints in late-stage disease. Prostate cancer is uncommon in men under 50. Weight loss and other systemic symptoms of disease such as bone pain and lethargy is associated with advanced metastatic disease and is otherwise uncommonly seen. Not all cases of malignancy classically features weight loss. Management of this condition is either expectant or definitive depending on life expectancy. Key investigations are PSA and DRE and diagnosis is confirmed on a trans-rectal ultrasound-guided needle biopsy. The normal prostate is about the size of a walnut and during examination, the size, consistency, symmetry and the presence of any nodules should be noted. A PSA normally ranges from 0-4 microgams/L however this varies with age and race. Prostate cancer is graded on the Gleason score (2-10), which is the sum total of two scales from 1 to 5 which is based on how differentiated the tumour’s predominant cell lines are. It is 95% of the time an adenocarcinoma as it is a malignant growth orginating from the glandular cells of the prostate.
A 35 year old lady presents with severe fatigue, malaise, nausea and joint pains worsening over the past few months. She has no other significant PMH apart from having had 3 first trimester miscarriages, She has a Hb of 7g/dl and creatinine of 280mmol/l.
What is the most likely diagnosis? A. Rheumatoid arthritis B. Nephrotic syndrome C. Prostate carcinoma D. Polycystic kidneys E. Bladder stones F. Bladder carcinoma G. Ureteric stones H. Glomerulonephritis I. Appendicitis J. SLE K. UTI L. Urinary tract obstruction M. Renal cell carcinoma N. Gout O. Acute tubular necrosis
J. SLE
This lady has systemic lupus erythematosus. SLE is a multi-system generalised disorder most commonly affecting women of reproductive age. This patient has associated antiphospholipid syndrome with a history of miscarriages and antiphospholipid antibiodies would be expected to be positive here. Lupus patients also commonly have renal manifestations and hence show elevated urea and creatinine. Anaemia may well be autoimmune here, with Coombs test being positive. SLE is characterised by the presence of ANA and most frequently involves the skin and joints. Most present with tiredness, the typical rash, which is not present here, and/or musculoskeletal symptoms like joint pain. Arthralgia is common and can be similar to RA but is non-erosive. The classic rash of lupus is the malar (butterfly) rash which presents as erythema over the cheeks and bridge of the nose, sparing the nasolabial folds. A photosensitive rash is classic, which occurs after sun exposure and there may also be a discoid rash which presents as erythematous raised patches with keratotic scaling. Symptoms and signs may accumulate over time.
An 80 year old woman who has been bed bound in hospital for a few months has become increasingly confused over the last two days and complains of abdominal tenderness.
What is the most likely diagnosis? A. Rheumatoid arthritis B. Nephrotic syndrome C. Prostate carcinoma D. Polycystic kidneys E. Bladder stones F. Bladder carcinoma G. Ureteric stones H. Glomerulonephritis I. Appendicitis J. SLE L. Urinary tract obstruction M. Renal cell carcinoma N. Gout O. Acute tubular necrosis
K. UTI
Confusion is not an uncommon presentation of UTI in the elderly. 10% of women >70 have a UTI. This is the cause of this elderly lady’s confusion. Her set of symptoms (confusion and abdominal tenderness) are not explained by any of the other conditions given on the list. A UTI, in uncomplicated cases is most commonly caused by E coli. However, this case could be a hospital acquired infection due to, for example, catheterisation and organisms such as Proteus and Pseudomonas are all possible. It is diagnosed with a dipstick and urine MC+S from an MSU sample. Have a think about what the dipstick would show. Antibiotic therapy should be guided by local sensitivities and guidelines, or MC+S results. Nitrofurantoin or co-trimoxazole could be used.
A 30 year old man was admitted for a severe infection a few days ago. His BP has now dropped, he has become tachycardic and his urine output has decreased. Blood test discloses a creatinine level of 800.
What is the most likely diagnosis? A. Rheumatoid arthritis B. Nephrotic syndrome C. Prostate carcinoma D. Polycystic kidneys E. Bladder stones F. Bladder carcinoma G. Ureteric stones H. Glomerulonephritis I. Appendicitis J. SLE K. UTI L. Urinary tract obstruction M. Renal cell carcinoma N. Gout O. Acute tubular necrosis
O. Acute tubular necrosis
Acute tubular necrosis is caused by either ischaemia or toxic injury to tubular epithelial cells in the kidney, which causes cell death or detachment from the BM. A history of low BP, fluid depeletion or toxin exposure is usually present. This patient has low BP and is fluid depleted, with compensatory tachycardia. This hypovolaemic state has resulted in ischaemic renal injury, which should have a good outcome once the hypovolaemia is corrected here with IV saline. The treatment is supportive.
A 73 year old woman is recovering in hospital 3 days after an operation to replace her right hip. On going to the toilet she suddenly becomes extremely short of breath and rapidly loses consciousness. Her BP is 60/20mmHg.
What is the most likely diagnosis? A. Pancreatitis B. Exacerbation of COPD C. Pulmonary Embolus D. Appendicitis E. Viral pneumonia F. Occulomotor (III) nerve palsy G. Diabetic amyotrophy H. GORD I. Abducent (VI) nerve palsy J. Pneumothorax K. Biliary colic L. Bowel obstruction
C. Pulmonary Embolus
This patient should be thrombolysed immediately if not contraindicated due to her haemodynamically unstable and critical state. Treatment should not be delayed in this obvious PE. This patient is now at a serious risk of cardiac arrest. Anticoagulation should also be started. The underlying pathophysiology is based on Virchow’s triad. SOB is a common symptom and there may also be chest pain and haemoptysis. This patient has had recent surgery, particularly orthopaedic surgery, which is a strong risk factor for PE. Other strong risk factors include DVT, obesity, prolonged bed rest, malignancy, previous VTE, pregnancy and the thrombophilias such as factor V Leiden. ECG may be normal, or may show tachycardia, new RAD, new RBBB or the classical S wave in I, Q wave with T inversion in III. Various clinical probability scores exist for PE and D-dimer can be used to exclude PE as a diagnosis.
The study of choice is a CTPA with direct visualisation of the thrombus. If there is a contraindication to a CT scan such as contrast allergy or pregnancy, then a V/Q scan is indicated. If a V/Q scan is not possible, alternatives such as MRA can be requested. It is worth noting that in patients with cardiopulmonary disease, these tests may not be accurate. A TTE can also be used to detect RV strain seen with PE.
A 77-year old retired coal miner has a 30-year history of cough, mostly productive of sputum. He suddenly becomes breathless after a bout of coughing and complains of right sided chest pain. On examination, he is cyanosed, the trachea is deviated to the left and no breath sounds are audible over part of the right side of the chest.
What is the most likely diagnosis? A. Pancreatitis B. Exacerbation of COPD C. Pulmonary Embolus D. Appendicitis E. Viral pneumonia F. Occulomotor (III) nerve palsy G. Diabetic amyotrophy H. GORD I. Abducent (VI) nerve palsy J. Pneumothorax K. Biliary colic L. Bowel obstruction
J. Pneumothorax
This patient has developed a right sided tension pneumothorax and will need emergency intervention in the form of the insertion of a large bore cannula into the 2nd intercostal space in the MCL of the affected side. This will need to be followed by the insertion of a chest drain.
Otherwise if this was not a tension pneumothorax, for secondary spontaneous pneumothoraces, if large enough for a chest drain or the patient is clinically unstable, chest drain insertion is indicated. Simple aspiration success rate is reduced in secondary spontaneous pneumothoraces. Primary pneumothoraces occur in young people without known lung conditions. This patient has pulmonary fibrosis. Those who suffer recurrent pneumothoraces may have to undergo pleurodesis to stick the parietal and visceral pleural together by an inflammatory reaction.
A 62 year old male diabetic presents with a sudden onset of double vision. He is noted to have ptosis and a deviation of the right eye down and to the right. The pupils appear equal in size and are reactive to light.
What is the most likely diagnosis? A. Pancreatitis B. Exacerbation of COPD C. Pulmonary Embolus D. Appendicitis E. Viral pneumonia F. Occulomotor (III) nerve palsy G. Diabetic amyotrophy H. GORD I. Abducent (VI) nerve palsy J. Pneumothorax K. Biliary colic L. Bowel obstruction
F. Occulomotor (III) nerve palsy
A complete (or surgical) third nerve palsy presents with compete ptosis, a dilated pupil and the eye turned down and out and can be caused by a PCA aneurysm. This is an emergency and the patient needs a neurosurgical opinion with MRI/angiography. A partial pupil-sparing (or medical) third nerve palsy presents with partial ptosis without pupillary signs and can be caused by infarction of the nerve i.e. mononeuritis multiplex, due to possible DM or vasculitis like GCA.
Remember from anatomy that CNIII innvervates all the mucles of the eye except SO and LR. CNIII also supplies levator palpebrae superioris and the sphincter pupillae which causes miosis. If the cause is surgical and compressive, the parasympathetic nerve is affected early on, as it lies at the surface of the third nerve. In an infarctive medical third nerve palsy, the centre infarcts, which is affected more than the surface of the nerve. Hence, the pupil is spared and there is partial ptosis.
A 4 year old boy, profuse vomiting, no motions for 2 days, severe abdominal pain which is intermittent in nature. Abdomen distended with increased bowel sounds – high pitched in nature.
What is the most likely diagnosis? A. Pancreatitis B. Exacerbation of COPD C. Pulmonary Embolus D. Appendicitis E. Viral pneumonia F. Occulomotor (III) nerve palsy G. Diabetic amyotrophy H. GORD I. Abducent (VI) nerve palsy J. Pneumothorax K. Biliary colic L. Bowel obstruction
L. Bowel obstruction
Bowel obstruction has several causes such as adhesions or cancer in the older patient. In children common causes of small bowel obstruction include appendicitis, intussusception, intestinal atresia and volvulus and these should be kept in mind in this case. The profuse vomiting, distended and painful abdomen and tinkling high pitched bowel signs are all indicative. The proximal segment of bowel dilates and distal bowel collapses. Completely obstructed patients generally require surgery. If, on AXR, air is seen to be seeping past the obstruction then the obstruction is partial. As a standard, all patients should be made NBM and given supplemental oxygen, IV fluids and NG decompression (to reduce flow/gastric contents/air towards the obstruction), unless they are rushed off for an emergency laparotomy because, for example, they have complete SBO and are peritonitic.